Download as pdf or txt
Download as pdf or txt
You are on page 1of 45

e

8/
NOVEMBER 2016 h,
ar
ig
nd

Multiple Choice Questions


ha

Answers & Explanations


IC
PG
PG
IC
ha
nd
ig
ar
h,
8/
e
NOVEMBER 2016  (MULTIPLE CHOICE QUESTIONS)

Anatomy d. Germ cell undergoes division to form diploid cell and in-
crease their number
1. Posterior interosseous nerve supplies all except: e. Body needs meiosis to produce large no. of eggs and
a. Extensor carpi radialis longus sperms
b. Extensor carpi ulnaris 9. Cross section through medulla at the level of mid-olivary
c. Extensor digitorum section through floor of fourth ventricle contains which of
d. Extensor indices the following structures:
e. Flexor carpi ulnaris a. Trapezoid body
2. Muscle(s) of anterior compartment of leg is/are: b. Dorsal nucleus of vagus
a. Peroneus tertius c. Nucleus of tractus solitarius
b. Peroneus brevis d. Nucleus ambiguus
c. Peroneus longus e. Superior vestibular nucleus
d. Flexor digitorum longus
e. Flexor hallucis longus

e
3. Branch(es) of Internal carotid artery directly arising from it: Physiology

8/
a. Posterior communicating artery
b. Superior hypophyseal artery 10. True statement(s) about Olfactory system :
c. Inferior hypophyseal artery a. Olfactory mucosa cover upper 1/3 of nasal cavity

h,
d. Posterior cerebral Artery b. Olfactory pathway passes via thalamus to orbitofrontal
e. Recurrent artery of Heubner ar cortex
4. All are true about vestibular nerve except: c. Adaptation to odour develop only after 1–2 minutes
a. It has two division- superior and inferior vestibular d. Olfactory receptors act via cAMP
e. Rate of olfactory nerve impulses change approximately in
ig
b. Vestibular nuclei situated at junction of pons and medulla
c. Nerve fibres relay at scarpa’s ganglion proportion to the logarithm of stimulus strength
11. True about special anatomy and physiology of lung:
nd

d. Nucleus lies in midbrain near aqueduct


5. True about Trochlear Nerve a. Surfactant prevent collapse of small alveoli into larger one
a. Arise from ventral aspect of brainstem b. Larger alveoli has more tendency to collapse than smaller
ha

b. Enters orbit through annulus of Zinn alveoli in absence of surfactant


c. Lesion causes diplopia c. Surfactant decrease chance of collapse
d. Nucleus of the trochlear nerve is located in the caudal d. Surfactant increases surface tension
IC

mesencephalon beneath the cerebral aqueduct e. With surfactant, large alveoli tend to become smaller and
e. Damage causes ipsilateral palsy of superior oblique muscle smaller ones tend to become larger
6. Content(s) of aortic hiatus: 12. All are true about acromegaly except:
a. Increased IGF-1 levels
PG

a. Thoracic duct b. Aorta


c. Vagus nerve d. Inferior vena cava b. Excessive growth occurs before fusion of the epiphyses of
e. Azygos vein the long bones
7. Which flexor tendon zone in hand is known as “No man’s c. Somatostatin analogues can be used
land”: d. Growth hormone levels increased
a. Zone I b. Zone II e. Transsphenoidal surgical resection is the preferred primary
c. Zone III d. Zone IV treatment for pituitary adenoma
e. Zone V 13. True statement(s) is/are:
8. Correct statement(s) about meiosis: a. Vasopressin increase only water reabsorption, not solute
a. Somatic cells not divide by meiosis because number of reabsorption
chromosomes reduces to half b. Aldosterone increase Na+ reabsorption from tubules
b. Occur in germ cell which result in haploid cells c. Glomerular filtrate of PCT has similar osmolarity as of
c. One spermatocyte produces one sperm and one oocyte plasma
produces one ovum d. Urine is hyperosmolar in early DCT
e. Generally urine osmolarity equals to plasma osmolarity

Answer Key
1. a, e. 2. a. 3. a, b, c. 4. d. 5. c, d. 6. a, b, e. 7. b.

8. a, b, e. 9. b, c, d. 10. a, b, d. e. 11. a, c, e. 12. b. 13. a, b, c.


PGI Chandigarh Self-Assessment & Review: 2017–2013

14. Rapidly adapting receptor(s) is/are: 22. Hyperphenylalaninemia occurs due to:
a. Pain receptor b. Pacinian corpuscles a. Phenylalanine hydroxylase deficiency
c. Muscle spindle d. Golgi tendon organs b. Phenylalanine hydroxylase overactivity
e. Meissner corpuscles c. Dihydrobiopterin reductase deficiency
15. Which of the following is/are true about normal blood level: d. Tyrosine hydroxylase deficiency
a. Total Calcium: 8.5–10.5 mg/dL e. Defect in dihydrobiopterin biosynthesis
b. Sodium:135–145 mmol/L 23. Oxidative phosphorylation not inhibited by:
c. Potassium: 3.5–5.1 mmol/L a. Fluoride b. 2, 4-dinitrophenol (DNP)
d. Creatinine: 0.6–2.6 mg/dL c. Oligomycin d. Carboxin
e. TSH level: 0.1–3.1 mIU/L e. Ouabin
16. True about Action potential in skeletal muscle fibers and 24. True about Apolipoproteins:
nerve fibres: a. Constitute peripheral region of plasma lipoproteins
a. Skeletal muscle fibres conduction velocity is ¼ of thick b. Divided into A, B, C only
myelinated nerve fiber c. Apo A-I is the major protein component of high density
b. Action potential of both qualitatively similar lipoprotein (HDL)
c. Resting membrane potential almost same d. Apo A,B and C are further divided
d. Duration of action potential same in both e. Role in enzyme activation

e
e. Action potential of both quantitively similar 25. Amino acids containing hydroxyl group:
a. Threonine b. Tyrosine

8/
17. Tissue elevation of which of the following cause vasocon-
striction : c. Serine d. Tryptophan
a. Na+ b. K+ e. Valine
26. Cytochrome P450 is/are involved in:

h,
c. Mg d. Ca2+
e. H+ a. Hydroxylation of xenobiotics
b. Methylation of xenobiotics
18. Which of the following statement(s) is/are true about
ar c. Deamination reaction
changes at time of ovulation:
d. Involved in hydroxylation of steroids
a. GnRH level decreases
ig
e. Drug interaction
b. Gonadotropin hormone surge
27. True about role of phospholipids:
c. hCG surge
nd

a. Cell to cell recognition


d. ↑ Prostaglandins
b. Cell signaling
e. Activation of proteolytic enzymes
c. Precursor of Second Messengers
19. Comprehension preserved in which of the following aphasia:
ha

d. Mediators of inflammation
a. Broca’s aphasia b. Conduction aphasia
e. Regulate membrane permeability
c. Wernicke’s aphasia d. Gobal aphasia 28. Correct statement about membrane :
IC

e. Anomic aphasia a. Phospholipids undergo rapid lateral diffusion


20. Vomiting centre(s) involved in post-operative vomiting: b. Transverse movement of lipids across the membrane is
a. Area postrema faster than protein
b. Chemoreceptor trigger zone (CTZ)
PG

c. Hydrophobic core of the phospholipid bilayer remains


c. Reticular formation located in medulla constantly in motion because of rotations around the
d. Nucleus tractus solitarius bonds of lipid tails
e. Basal ganglia d. Phospholipds that have one fatty acyl group, cannot form
the bilayer
e. Phospholipids span whole bilayer
Biochemistry
21. Hyperuricemia is/are associated with: Immunogenetics &
a. HGPRT deficiency Molecular Biology
b. HGPRT overactivity
c. PRPP synthetase deficiency 29. During DNA replication which bond breaks:
d. Glucose 6- phosphatase deficiency a. Phosphodiester bonds b. Phosphate bond
e. Glucose phosphate dehydrogenase deficiency c. Hydrogen bond d. Glycosidic bonds
MCQs

Answer Key
14.
b, d, e. 15. a, b, c. 16. b, c. 17. d. 18. b, d, e. 19. a, b, e. 20. a, b, d.
21. a, d. 22. a, c, e. 23. a, d, e. 24. a, c, d, e. 25. a, b, c. 26. a, c, d, e. 27. b, c, d, e.

28. a, c, d. 29. c.

136
November  | 2016

30. Which RNA contains abnormal purine and pyrimidine: 38. Feature(s) of Adult polycystic kidney disease is/are:
a. tRNA b. 23SrRNA a. Renal enlargement
c. 16SrRNA d. 5SrRNA b. Small kidney
e. mRNA c. Spider leg deformity on intravenous urography
31. Component of 50S ribosomal subunit: d. Ultrasound shows multiple cysts
a. 16S RNA b. 18S RNA 39. Which is/are caused by protein misfolding:
c. 5.8 RNA d. 5S RNA a. Creutzfeldt–Jakob disease
e. 23S RNA b. Bovine spongiform encephalopathy
32. True about Chromatin remodeling: c. Huntington disease
a. Energy is required to displace the histone octamers from d. Alzheimer disease e. Parkinson disease
DNA or translocate them onto neighboring DNA segments 40. True about Creutzfeldt-Jakob Disease :
b. Histone modifications by specific enzyme a. Gliosis in thalamus
c. Do not involve enzymes b. Spongiform swelling in cerebral cortex
d. Aberrations in chromatin remodeling proteins may be c. Brain atrophy in late stage
associated with cancer d. Slow and irregular background rhythm on EEG
33. Best assessment of protein binding regions on a DNA mol- 41. Hereditary non-polyposis colorectal cancer (HNPCC) is/are
ecule can be done by: commonly associated with:

e
a. DNA footprinting b. RT PCR a. Endometrial cancer b. Cervical cancer
c. Ovarian cancer d. Breast cancer

8/
c. Microarray d. Western blotting
e. Northern blotting e. Thyroid cancer
42. Feature of Non Bacterial thrombotic endocarditis(NBTE):

h,
a. Common in SLE
Pathology b. Present on undersurface of valve
c. Vegetative growth is large and loosely attached to valve
34. Feature(s) of Chronic myelogenous leukemia (CML):
ar d. May occur after post-cardiac catheterization
a. Bone narrow biopsy is necessary for diagnosis e. Source of systemic emboli
ig
b. Presence of BCR-ABL gene which directs the synthesis of 43. True about Alzheimer disease:
BCR-ABL tyrosine kinase a. Most common cause of dementia in elderly
nd

c. Dasatinib is used in imatinib resistant cases b Unusal before 45 years of age


d. Generalized painful lymphadenopathy is presenting fea- c. Plaques consists of tau protein
ture in most cases d. May have family history
ha

e. Myeloblasts usually constitute more than 10% of all white e. Short term memory is affected less than long-term memory
cells in chronic phase 44. Negri bodies in animal can be best seen in:
35. True about Cystic fibrosis: a. Hippocampus b. Basal ganglia
IC

a. Occurs due CFTR gene mutation on chromosome 7 c. Cerebral cortex d. Cerebellum


b. Meconium ileus is present in >90% cases e. Thalamus
c. CFTR gene can be detected antenately 45. True about p53 :
PG

d. Poor body growth a. Has tyrosine kinase activity


36. True about caspases: b. Has pro-apoptotic activity
a. Caspases initiate apoptosis by extrinsic and intrinsic path- c. Tumour suppressor gene
way d. Has anti-apoptotic activity
b. Caspases are protease enzyme 46. True about Takayasu syndrome:
c. Caspases are receptor a. Involves small and Medium sized vessels
d. Caspases inhibit apoptosis b. Shares many clinical features of giant cell arteritis if
e. Causes non enzymatic degradation of critical cellular involves aorta
components c. More common in male than female
37. True about minimal change disease: d. Granulomatous vasculitis
a. Hypertension is commonly present e. Also called pulseless disease
b. Most common cause of nephrotic syndrome in adults 47. Which are inheritable malignancies:
c. High dose steroids results in remission in most cases a. Breast cancer b. Thyroid cancer
d. Commonly progress to chronic renal failure c. Wilm’s tumour d. Retinoblastoma MCQs
e. Reversible loss of podocyte function e. Prostate cancer

Answer Key
30.
a. 31. d, e. 32. a, b, d. 33. a. 34. a, b, c. 35. a, c, d. 36. a, b.
37. c, e. 38. a, c, d. 39. All (a, b, c, d, e) 40. b, c, d. 41. a, c. 42. b, d, e. 43. a, b, d.

44. a, d. 45. a, b, c. 46. b, d, e. 47. All (a, b, c, d, e)

137
PGI Chandigarh Self-Assessment & Review: 2017–2013

48. Cause(s) of Non-megaloblastic macrocytic anemia: 56. Which of the following is/are newer drugs for TB:
a. Folate deficiency a. Bedaquiline
b. Lead toxicity b. Clofazimine
c. Hypothyroidism c. Coftaroline
d. Liver disease d. Rifapentine
e. Vit B12 deficiency e. Etanercept
49. True statement(s) about Wilm’s tumor: 57. All are true about oral iron therapy in anemia except:
a. Most commonly presents as asymptomatic abdominal a. May worsen inflammatory bowel disease
mass b. It takes minimum 2 weeks for reticulocyte count to
b. Hereditary predisposition is present in 50% cases increase
c. Bilateral in 25% cases c. Generally 3-6 month therapy is required to replenish iron
d. Classic triphasic combination of blastemal, stromal, and stores
epithelial cell types is observed d. Gastrointestinal side-effects limits its dose
e. Most common in children e. Hb level is generally attained in 1-3 month
50. Which of following is not classified as Primitive neuroecto- 58. Drug which can be given by inhalation route:
dermal tumour (PNET): a. Zileuton
a. Retinoblastoma b. Medulloblastoma b. Steroid

e
c. Rhabdomyosarcoma d. Ewing sarcoma c. Salbutamol

8/
e. Carcinoid tumour d. Tobramycin
51. Glucose level in CSF is/are reduced in: 59. Liver function test (LFT) monitoring is/are required in use
a. Bacterial meningitis b. Fungal meningitis of which of the following Disease Modifying Antirheumatic

h,
c. Viral meningitis d. Tubercular meningitis Drugs (DMARDs):
e. Spirochetal meningitis ar a. Methotrexate b. Hydroxychloroquine
52. True about proliferative phase of wound healing c. Sulfasalazine d. Leflunomide
a. Neutrophils increases gradually e. Gold
b. Macrophage increases gradually 60. Drugs which can be used in gestational hypertension:
ig
c. Collagen type I present predominantly a. Metoprolol b. Labetalol
d. Collagen type III present predominantly c. Methyldopa d. Sustained release nifedipine
nd

e. Angiogenesis occurs e. Losartan


53. Correct match of stain with tissue is/are: 61. Adverse effects of mirtazapine is/are:
a. Perls’ Prussian blue-iron in tissue a. Insomnia b. Sedation
ha

b. Von Kossa-collagen c. Sexual dysfunction d. Vomiting


c. Masson’s trichrome –elastin fiber e. Weight gain
IC

d. PAS- glycogen 62. Carbonic anhydrase inhibitor(s) is/are:


e. PAS- Acidic and neutral mucin a. Acetazolamide
b. Amiloride
c. Nitrofurantoin
PG

Pharmacology d. Topiramate
63. Which of the following is/are true about Tacrolimus:
54. Which of the following is/are action of estrogen except: a. A macrolides antibiotic
a. Development of the alveolar system b. Structure similar to cyclosporine
b. Proliferation of stroma of breast c. Derived from a fungus
c. Ductal growth of the breast d. T cell inhibitor
d. ↓ Bone resorption e. Hirsutism less evident than cyclosporine
e. Development of lobules of breast 64. True about Low molecular weight heparin (LMWH):
55. All are true about bisphosphonates except: a. Anti-factor Xa assay monitoring required in every patient
a. Prevent reabsorption of bone by osteoclast b. It increases aPTT more than UFH
b. Structurally similar to pyrophosphate c. Can be safely given in renal failure
c. Absorption increases with food d. Toxicity is totally reversed by protamine sulphate
d. Can be safely given in liver disease e. Inactivate factor Xa selectively
MCQs

Answer Key
48.
c, d. 49. a, d, e. 50. a, c, d, e. 51. a, b, d. 52. d, e. 53. a, d, e. 54. a, e.
55. c. 56. a, b, d. 57. b. 58. b, c. 59. a, d, e. 60. a, b, c, d. 61. b, e.

62. a, d. 63. a, d, e. 64. e.

138
November  | 2016

65. True about Mafenide: 72. Drug that can potentiate Torsades de pointes:
a. Can penetrate eschars a. Amiodarone b. Sotalol
b. Doesn’t cause burning sensation when applied to raw c. Chlorpromazine d. Cisapride
surface e. Aspirin
c. Can be used orally
d. May cause metabolic acidosis
66. Which of the following is/are true about pharmacodynamics Microbiology
of drugs:
a. Affinity means how strongly drug binds to receptor 73. All is/are true about Rotavirus infection except:
b. Efficacy means maximal effect by a drug a. Most commonly seen in adult of > 30 year age group
c. Irreversible antagonist mainly forms ionic bonds with b. Person to person transmission may occur
receptor c. Severity of disease decreases with each repeat infection
d. Agonist potency depends on two parameters: affinity and d. Commonest cause of diarrhea in infants and children
efficacy e. Single infection provide lifelong immunity against reinfec-
e. For antagonists, efficacy is zero tion
67. All are true about plasma protein binding except: 74. True about pseudomonas aeruginosa :
a. Acidic drugs generally bind to plasma albumin and basic a. Not lysine decarboxylase positive

e
drugs to a1 acid glycoprotein b. Oxidase positive
c. Produce pyocyanin pigment

8/
b. Plasma binding determines volume of distribution
c. More plasma protein binding means more storage in liver d. Gram-negative bacilli
d. More plasma protein binding means less penetration in e. Has 6–12 flagella
75. Which of the following is/are DNA viruses:

h,
vascular membrane
e. High degree of protein binding generally makes the drug a. Herpes virus b. Hepadnaviridae
c. Parvovirus d. Orthomyxoviridae
long acting
ar e. Enteroviruses
68. First pass metabolism is significant problem in drug given
through: 76. True about ZIKA virus:
ig
a. Sublingual route a. Belong to flavivirus
b. Rectal route b. First case detected in 1953 in Nigeria
nd

c. Intramuscular route c. RT PCR is useful in diagnosis


d. Directly into stomach d. Causes macrocephaly
e. Directly into large intestine e. May presents with conjunctivitis
ha

69. Which of the following dyads show clinically significant 77. Parasite which infects through ingestion of aquatic
drug interactions : vegetation:
a. Fasciola hepatica
IC

a. Vancomycin—Amphotericin B
b. Rantidine– Atorvastatin b. Fasciolopsis buski
c. Warfarin— Aspirin c. Paragonimus westermani
d. Allopurinol— Azathioprine d. Watsonius watsoni
PG

e. Aminoglycoside+Vancomycin e. Gastrodiscoides hominis


70. True about osmotic diuretics: 78. Which of the following dyads of vector with disease is/are
a. Osmotic diuretics have their major effect in the distal correctly matched:
convoluted tubule a. Rat flea– Endemic typhus
b. Contraindicated in congestive heart failure b. Sand flea—Oriental sore
c. Causes Hyperkalemia c. Black fly—Kala azar
d. Increases renal blood flow d. Cyclops—Dracunculosis
71. True about effect of steroid intake in inflammatory condi- e. Louse—Chagas disease
tions: 79. All are feature of Scrub typhus except:
a. Proanabolic effect on muscles a. Black eschar
b. ↑ glucose in plasma b. Maculo-papular rash
c. –ve feedback on corticotropin-releasing hormone (CRH) c. More common in rural areas
production d. Ciprofloxacin is drug of choice
d. May cause osteoporosis e. Tick borne disease MCQs

Answer Key
65.
d. 66. a, b, d, e. 67. c. 68. d, e. 69. a, c, d, e. 70. b, c, d. 71. b, c, d.
72. a, b, c, d. 73. a, e. 74. a, b, c, d. 75. a, b, c. 76. a, c, e. 77. a, b, d, e. 78. a, b, d.

79. d, e.

139
PGI Chandigarh Self-Assessment & Review: 2017–2013

80. Unlike nocardia, Actinomyoses is: 88. All are true about Dengu virus except:
a. Facultative anaerobes a. Belong to flaviviridae
b. Not acid fast b. Type DEN 4 is most common in India
c. Endogenous cause of disease c. Main vectors are aedes aegypti and aedes albopictus
d. Environmental saprophyte d. Virus has positive sense RNA
e. Grow at wide range of temperature range e. Vector is sensitive to DDT
81. Which of the following is/are caused by dermatophytes: 89. Which of the following is/are true about Dengu fever:
a. Madura foot b. Athlete’s foot a. Positive Tourniquet test means more than 10 petechiae per
c. Kerion d. Favus square inch
82. Which of the following has least minimum infective b. Caused by flavivirus
dose(MID) required for causing infection: c. Aedes aegypticus and albopictus are most important vector
a. Salmonella typhi b. Campylobacter jejuni in India
c. Shigella dysentery d. Vibrio cholera d. IgM/IgG ratios may be used to distinguish primary from
83. Which type of bacteria can not survive in absence of oxygen: secondary infection
a. Obligate aerobe b. Facultative anaerobes e. No vaccine available at present
c. Microaerophilic d. Obligate Anaerobes 90. Autoclave is/are used for sterilization of:
e. Facultative aerobes a. Wooden material b. Metallic instrument

e
84. All are true about H1N1 influenza except: c. Plastic d. Glasswares

8/
a. Zanamivir commonly given through IV route e. Fibro-optic bronchoscope
b. Fatality more in some high risk group
c. RT-PCR is used for investigation

h,
d. WHO latest trivalent influenza vaccine contains two Forensic Medicine
influenza A subtypes (H3N2 and H1N1) and one influenza
91. Valid ground of divorce for female:
B component
ar
a. Impotence of male partner
e. CDC latest quadrivalent influenza vaccine contains two
influenza A subtypes (H3N2 and H1N1) and two influenza b. Extramarital affair of male aprtner
ig
B component c. Infertility of female
85. True about serum marker of inactive carrier phase of chron- d. Unemployment of male partner
nd

ic Hepatitis B: e. If she was married before the age of fifteen and she want to
a. Hbs Ag +ve renounces the marriage before she attains eighteen years of
b. Hbe Ag +ve age
ha

c. Anti-HBe antibody positive 92. Bite mark may aid in:


d. Low level DNA a. Assessment of age
b. Identification of accused
IC

e. Increased ALT
86. Which of the following mechanism is/are used by bacteria to c. Assessment of height
escape host defence mechanism: d. Assessment of type of teeth
a. Mycobaterium tuberculsosis prevent intracellular killing e. Collection of DNA sample
PG

by inhibiting phagolysosome formation 93. Feature of gunshot injury is/are:


b. Streptococcus pyogenes by M protein a. Gutter injury
c. Neisseria meningitides by capsular polysaccharide b. Grease collar
d. Staphlococcus aureus by iron-regulated outer membrane c. Radiating wound
proteins d. Tissue opening on opposite end
e. Polysaccharide capsules of H. influenzae 94. Which of the following is/are true regarding Perjury:
87. True about Gas gangrene: a. Wilfully giving false statement under oath which he/she
a. Onset is usually acute either knows or believes to be false or does not believe to
b. Painless condition be true
c. Wound is swollen b. S. 190 IPC deals with perjury
d. At first wound is dusky or red, later becomes pale c. Voluntarily giving false evidence under oath which he/she
e. Caused by gram +ve organism either knows or believes to be false or does not believe to
be true
MCQs d. The witness is liable to be prosecuted for perjury

Answer Key
80.
a, b, c. 81. b, c, d. 82. c. 83. a. 84. a. 85. a, c, d. 86. a, b, c, e.
87. a, c, e. 88. b. 89. All (a, b, c, d, e) 90. b. 91. a, b, e. 92. b, d, e. 93. All (a, b, c, d)

94. a, c, d.

140
November  | 2016

95. True about Corpus delicti : 102. Which of the following content is/are more in Human milk
a. Medical negligence than cow’s milk:
b. Body of offence a. Protein b. Iron
c. It includes body of the victim and other facts which are c. Carbohydrate d. Fat
conclusive of death by foul play e. Energy
d. The essence of crime 103. Which of the following is/are true :
96. Which is not a method of crime scene examination: a. Serial interval = gap in time b/w invasion by an infectious
a. Grid b. Strip agent and appearance of clinical feature
c. Wheel d. Composite b. Latent period = the period from disease initiation to
e. Point to point disease detection in non-infectious disease
c. Incubation period = time b/w the onset of the primary case
and the secondary case
Preventive & Social Medicine d. Generation time = time period between the onset of the
infection and the maximum infectivity of the host
97. Which of the following is/are true about tuberculosis in
e. Communicable period = It is a period during which the
India:
reservoir is infectious to others
a. India has approximately 1/4th of Global load of TB
104. Tests to check pasteurization of milk:

e
b. MDR-TB among notified new pulmonary TB patients is
a. Phosphatase test b. Standard plate count
about 5%

8/
c. Methylene blue test d. Nitric acid test
c. 5% of TB patients estimated to be HIV positive
e. Coliform count
d. MDR-TB among retreatment cases is about 15%
e. Incidence is around 2 million new TB cases annually

h,
98. Importance of lepromin test are all except:
a. Only has epidemiological significance
ar Ophthalmology
b. Prognostic value 105. True about component of vision 2020:
c. Tells about immunity status of leprosy patients a. Cataract surgery should be performed at primary level
ig
d. Differentiate between different types of leprosy b. Retinal surgery should be performed at tertiary level
e. Predictive value c. Need to develop 10 centre of excellence at tertiary level and
nd

99. A leprosy person is presented with involvements of sural 100 training centre at advanced tertiary level
and radial nerve. Which type of regimen you will give: d. Ophthalmia neonatorum is included in childhood blind-
a. Multibacillary treatment × 9 month ness
ha

b. Multibacillary treatment × 12 month e. Primary vision center covers a population of 50000


c. Multibacillary treatment × 15 month 106. True about Nodular episcleritis:
d. Paucibacillary treatment × 6 month a. Can be associated with SLE
IC

e. Single dose treatment of Rifampicin, Ofloxacin and b. Take longer time to resolve than diffuse variety
Minocycline (ROM) c. More symptomatic than diffuse type
100. Which of the following is true about data representation: d. Painless
PG

a. Histogram is used for presentation of discrete data e. Elevated hard nodule


b. Random dots in scatter diagram— no correlation 107. Which type of cataract is/are associated with Myotonic
c. Pictogram is represented by small pictures or symbols dystrophy:
d. Pie chart is represented by quadrangular figures a. Posterior subcapsular
e. Regression graph is said linear when the increase or b. Anterior subcapsular
decrease in the variables remains proportional in different c. Nuclear cataract
subjects d. Cortical cataract
101. True about Program evaluation and review technique 108. What is the WHO criteria for defining blindness:
(PERT): a. <3/60 vision with available correction
a. Better than critical path method for small project b. <6/60 vision with available correction
b. Recurrent activities is better monitored than CPM c. <3/60 vision with best correction
c. Activities are divided into small goals d. <6/60 vision with best correction
d. Main objective to monitor cost e. <3/60 vision without correction
e. It is a management technique better for non-research MCQs
activities than CPM

Answer Key
95.
b, c, d. 96. d, e. 97. a, c, d, e. 98. a. 99. b. 100. b, c, e. 101. c.
102. b, c. 103. b, d, e. 104. a, b, c, e. 105. b, d, e. 106. a, b, c, e. 107. a. 108. c.

141
PGI Chandigarh Self-Assessment & Review: 2017–2013

109. True about Acanthamoebic keratitis: 115. All are true about epistaxis except:
a. More in those wearing rigid gas permeable than soft a. Keisselbach’s plexus is source in 90% cases
contact lenses user b. If anterior packing is left in nose for more than 48hrs
b. Can occur as opportunistic infection in patients with antibiotic coverage is given
herpetic keratitis c. Anterior nasal pack is easy to insert and less traumatic
c. Can be cultured on non-nutrient agar enriched with E. coli than balloon temponade
d. Painful condition d. Trotter method is first aid method
e. Radial kerato-neuritis may occur e. Cauteristaion is done in refractory cases under general
110. True about ferritin line in eye: anaesthesia
a. Ferry’s Line-corneal epithelial iron line at the edge of 116. True about Secretory otitis media:
filtering blebs a. Type C tympanogram may be seen in early stage of otitis
b. Stockers Line-Corneal epithelial iron line at the edge of media with effusion
pterygium b. Flat tympanogram is present
c. Hudson-Stahli line- visible all around the base of cone in c. Leads to conductive deafness
Keratoconus d. Presence of cleft palate reduces its chance
d. Fleischer’s ring: Horizonatl corneal epithelial iron line at e. Most common cause is Eustachian tube dysfunction
the inferior one third of cornea due to aging 117. Which cause reddish lesion on tongue:

e
e. Coat’s white ring- A form of iron deposit at the level of a. Median rhomboid glossitis

8/
Bowman’s layer of cornea b. Hairy leukoplakia
111. Which of the following is the feature of sodium fluorescein c. Lichen planus
angiography compared to indocyanine green angiography: d. Geographic tongue

h,
a. In choroidal circulation it passes freely across the e. Fordyce’s spots
endothelium ar118. True statement regarding wax in ear:
b. Diffuse freely through retinal capillaries a. Syringing and instrumental manipulation are generally
c. Albumin binding is less than indocyanine green done to remove impacted wax
d. Bind < 50% to albumin b. If wax is hard and impacted, ceromiilytic substances is
ig
e. Stimulated by a longer wavelength of light used to soften wax
112. True about phthisis bulbi: c. In syringing fluid is injected along the lower wall of the
nd

a. Size of eye decreases meatus


b. Removed by enucleation operation especially if painful d. Wax has antibacterial property
ha

c. IOP increases in late stage 119. True about spasmodic dysphonia:


d. Calcification may occur in Bowman’s layer of cornea a. A neurological problem
113. Advantage of continuous curvilinear capsulorhexis over can b. Mostly psychogenic in origin
IC

opener technique : c. Hyperadduction of vocal cord may be seen


a. Preferred method of anterior capsulotomy in phacoemul- d. Botulinum toxin relives spasm
sification e. Speech therapy is beneficial
b. More chances of posterior capsular opacification 120. Feature(s) of peritonsillar abscess:
PG

c. Keeps the nucleus in place a. Foul breath


d. Resists radial tears b. Hot potato voice
e. Helps in stabilizing and centering the lens implant c. Shifting of uvula in opposite side
d. Difficulty in swallowing even own saliva
e. Always presents as b/l severe pain in throat
ENT 121. True about antrochoanal polyp:
a. Starts as edema of maxillary sinus mucosa
114. All is true about foreign body impaction in ear except: b. Suppressed by steroids
a. Objects located medial to isthmus of canal is difficult to c. Comes out via accessory ostium and grows in the choana
remove and nasal cavity
b. Syringing is used for removal of vegetative foreign body d. More common in adults than children
c. Syringing uses room temperature water e. Commonly presents as unilateral nasal obstruction
d. Blunt hook is used to remove rounded foreign body
MCQs e. GA is preferred in children to remove foreign bodies

Answer Key
109.
b, c, d, e. 110. a, b, e. 111. a, c. 112. a, b, d. 113. a, c, d, e. 114. b. 115. c.
116. a, b, c, e. 117. a, d. 118. a, b, d. 119. a, c, d, e. 120. a, b, c, d. 121. a, c, e.

142
November  | 2016

Medicine 130. True about Hypersensitivity pneumonitis:


a. Occurs due to inorganic antigen
122. Which of the following is/are feature of aortic stenosis: b. Increased CD8+ T cells in bronchoalveolar lavage
a. Duration between onset of symptom and death is generally c. Manifests mainly as an occupational and environment
10–15 year disease
b. Angina occurs mainly because of fixed coronary blood flows d. For severe acute cases, oral steroids is given for 3–4 weeks
c. No increase in cardiac output despite exercise e. Interstitial inflammatory infiltrate is seen in lung
d. Ejection systolic murmur radiating to neck may present 131. All are true about Sjögren’s syndrome except:
e. Left ventricular hypertrophy a. Bilateral parotid gland enlargement
123. True about organophosphate-induced delayed polyneurop- b. Parotid gland enlargement may be painful
athy: c. Xerostomia may present
a. Usually occurs after 2–3 month of acute exposure d. Progression to lymphoma
b. Involves both sensory and motor nerve e. Males are affected more than females
c. Steroid is used for treatment 132. Feature(s) of Felty’s syndrome is/are:
d. Incomplete recovery a. Seropositive for rheumatoid factor
124. Unlike Child Pugh scoring, MELD score have: b. Splenomegaly
a. Bedside assessment easy c. Long standing rheumatoid arthritis

e
b. Prothrombin time expressed as international normalized d. Neutrophilia
ratio (INR)

8/
e. Keratoconjunctivitis sicca
c. Serum creatinine estimation 133. Malignancies associated with AIDS:
d. Four component is used in scoring a. Primary CNS lymphoma

h,
e. Albumin level estimation b. Cervical cancer
125. Risk factors associated with health care associated pneumo- c. Kaposi sarcoma
nia (HCAP):
a. Acute care hospitalization for at least 2 days in the preced-
ar d. Ovarian cancer
e. Endometrial cancer
ing 90 days 134. Malignancy associated with hypercalcemia:
ig
b. Home infusion therapy a. Breast cancer b. Small cell lung cancer
c. Immunosuppressive disease or immunosuppressive c. Non-small lung cancer d. Prostate cancer
nd

therapy e. Multiple myeloma


d. Antibiotic therapy in the preceding 90 days 135. True about Severe Combined Immunodeficiency (SCID):
e. Hospitalization for ≥ 48 h a. Adenosine deaminase enzyme may be given for treatment
ha

126. All are feature(s) of sarcoidosis except: b. Haematopoietic stem cell transplant (HSCT) is curative
a. High CD4: CD8 ratio c. Most common inheritance is X linked recessive and
b. Hypercalciuria and hypercalcimia may be present
IC

autosomal recessive
c. ↑ Serum levels of angiotensin-converting enzyme (ACE) d. Lymphocytosis is present in most cases
d. Schauman and asteroid bodies are pathognomic e. Increased risk of infection by pneumocystis jiroveci
127. Transudative pleural effusion occurs in: 136. Proximal renal tubular acidosis(RTA) is/are associated with:
PG

a. Urinothorax b. Dressler syndrome a. Fanconi anemia b. Multiple myeloma


c. Nephrotic syndrome d. Myxedema c. Lead poisoning d. Sjogren’s syndrome
e. Congestive heart failure e. SLE
128. True about primary sclerosing cholangitis: 137. High anion gap acidosis is/are associated with:
a. Involves only intrahepatic bile duct, not extrahepatic bile a. Lactic acidosis
duct b. Ethylene glycol poisoning
b. Associated with Inflammatory bowel disease c. Aspirin overdose
c. Causes macronodular cirrhosis d. Diarrhea
d. Periductal fibrosis of smaller bile ducts e. Renal tubular acidosis
129. True about insulinoma: 138. Tubular proteinuria is/are seen in :
a. Encapsulated a. Multiple myeloma
b. Mostly multiple b. Wilson disease
c. Associated with MEN-I c. Lead poisoning
d. Enucleation is the treatment of choice for benign tumour d. Fanconi syndrome
MCQs
e. Histology similar to normal β-cells
Answer Key
122.
c, d, e. 123. a, b, d. 124. b, c. 125. All (a, b, c, d, e) 126. d. 127. a, c, d, e. 128. b, d.
129. a, c, d, e. 130. b, c, d, e. 131. e. 132. a, b, c, e. 133. a, b, c, d. 134. a, c, d, e. 135. a, b, c, e.

136. a, b, c. 137. a, b, c. 138. b, c, d.

143
PGI Chandigarh Self-Assessment & Review: 2017–2013

139. All are true about Celiac disease except: 147. True about idiopathic thrombocytopenic purpura:
a. Antiendomysial antibody is present a. In children, it is usually an chronic disease
b. Oat, rye and barley can be safely given b. Self-limited course in acute form
c. Associated with dermatitis herpetiformis c. In adults, it is a more acute disease
d. Associated with gliadin d. Immune-mediated destruction of platelets
e. No risk for development of cancer 148. True about coagulation disorders:
140. Respiratory failure type II is/are seen in: a. In DIC both PT and aPTT increase
a. Myasthenia gravis b. Hemophilia C is a X linked recessive condition
b. Acute exacerbation in COPD c. Factor VIII can be given in hemophilia B
c. Acute severe asthma d. Hemophilia A is inherited as X-linked recessive
d. Pulmonary edema 149. Plexiform lesion is prominent in which group of pulmonary
e. Pulmonary embolism hypertension:
141. Photosensitivity is/are not seen in : a. Recurrent thromboemboli
a. Acute intermittent porphyria b. Interstitial lung diseases
b. Variegate porphyria c. Familial pulmonary HTN
c. Porphyria cutanea tarda d. Congenital heart disease with left-to-right shunts
d. Congenital erythropoietic porphyria e. Pulmonary hypertension associated with human immuno-

e
e. Erythropoietic protoporphyria deficiency

8/
142. True about 4th heart sound: 150. In which of the following condition, non-hepatic surgery is
a. Low pitch associated with most adverse outcome:
b. Present during early diastole a. Child-Pugh score B

h,
c. Absent in atrial fibrillation b. Child-Pugh score C
d. Produced in the ventricle during ventricular filling phase ar c. Acute viral hepatitis
e. Present in severe left ventricular hypertrophy d. Acute alcoholic hepatitis
143. True about atrial flutter: e. Chronic viral hepatitis
a. Narrow-complex tachycardia of up to 150/min 151. True about multiple sclerosis:
ig
b. P wave absent a. Periventricular involvement can’t be seen by imaging stud-
c. Associated with 2 : 1, 3 : 1 or 4 : 1 AV block ies
nd

d. Best therapy is catheter ablation b. May produce mass lesion


e. Occur due to macro re-entry circuit within the right c. Autoimmune inflammatory condition
atrium d. Oligoclonal bands may be present in CSF
ha

144. Positive ECG sign(s) of ischemia in Tread mill test is/are: e. Spinal cord involvement may occur
a. Upsloping depression of the ST segment >0.1 mV below 152. Safest transplantation approach in liver disease :
IC

baseline a. Directly transplanting embryonic stem cell in the liver


b. Downsloping depression of the ST segment >0.1 mV below b. Transplanting donor hepatocytes into liver
baseline c. Transplanting mesenchymal stem cell from adipose tissue
c. Junctional ST-segment to liver
PG

d. Tachycardia d. Injecting erythropoietin into body


e. Ventricular premature beats
145. Which of the following is/are included in management of
acute ischemic stroke: Surgery
a. Unfractioned Heparin b. LMWH
c. Streptokinase d. Aspirin 153. True about postoperative ileus:
e. Recombinant tissue plasminogen activator (rt-PA) a. Colon recovers earlier than small intestine
146. Finding(s) in hemolytic anemia is/are: b. Small intestine most commonly affected
a. Increase in conjugated bilirubin c. Adhesion is most common cause
b. Increase in unconjugated bilirubin d. Usually resolves within 48–72 hour
c. Increase in urine urobilinogen e. Radiographs show diffusely dilated bowel with air in the
d. Increase in faecal stercobilinogen colon and rectum
e. Increased bilirubin in urine
MCQs

Answer Key
139.
b, e. 140. a, b, c. 141. b. 142. a, c, d, e. 143. a, c, e. 144. b. 145. d, e,
146. b, c, d. 147. b, d. 148. a, d. 149. c, d, e. 150. b. 151. b, c, d, e. 152. c,

153. d, e.

144
November  | 2016

154. True about secondary bacterial peritonitis: 161. Non-absorbable suture(s) is/are:
a. Marked leukocytosis with left shift of the WBCs to band a. Silk
forms b. Catgut
b. Usually caused by polymicrobial infection c. Polypropylene
c. May be associated with appendicular perforation d. Polyester
d. Associated with cirrhosis of the liver e. Nylon
155. Correct statement about intestinal ischemia: 162. Correct statement(s) about surgical site infection:
a. Watershed zones are usually affected a. Can occurs due to environmental flora in OT
b. Arterial ischemic lesion has better demarcation than b. Superficial infection require re-surgical exploration of
ischemia caused by impaired venous drainage wound
c. Crypts may be hyperproliferative c. Does not occur later than 14 days post-operative
d. Microscopic examination may show coagulative necrosis d. Shaving is beneficial for prevention
of the muscularis propria e. Can occur even after 1 yr in case of implant left in situ
e. Surface epithelium is normal 163. Regarding pre-operative antibiotic prophylaxis, which of
156. True about Colorectal cancer: the following statement(s) is/are correct:
a. Right side cancer usually presents with obstructive a. Should be started 2 days before surgery
symptoms b. Should be given half an hour prior to surgery

e
b. May be associated with HNPCC c. Not needed in clean surgery

8/
c. Left-sided cancer presents with alteration in bowel habit d. Should be given more than 1 hour before surgery
d. Left side colon is more commonly involved 164. Grade III (contaminated) wound is/are
157. True about thyroid cancer: a. Appendicular perforation

h,
a. Follicular cancer is more common in radiation exposed b. Wound contaminated with gross fecal material spillage
patient
ar c. Wound in urinary tract without unusual contamination
b. Hürthle cell carcinoma has better prognosis than follicular d. Surgery over a clean site
cancer e. Incision through abscess
c. Follicular cancer have better prognosis than pupillary 165. True about testicular cancer:
ig
cancer a. Seminoma is more common than non-seminoma
d. Insular variant of papillary carcinoma has better prognosis b. Cryptorchidism is associated with seminoma
nd

than papillary cancer c. Retroperitoneal LN dissection is done for grade II testicular


e. Medullary carcinoma is associated with MEN-2b tumour
158. True about Schatzki ring: d. Usually presents as painless testicular lump
ha

a. Contain true esophageal muscle e. Seminomas metastasise mainly via haematogenous route
b. Concentric symmetric narrowing of lower esophagus 166. True about obstructive jaundice :
IC

c. Consists of esophageal mucosa above and gastric mucosa a. Cholangiocarcinoma cause intermittent jaundice
below b. Courvoisier law related to pancreatic head cancer
d. Dysphagia is usually to solid foods c. Cholangitis presents with fever and jaundice
e. Association with reflux disease d. Increase in urine urobilinogen
PG

159. True about intestinal type of gastric cancer: e. Increase in fecal urobilinogen
a. Incidence decreased nowadays 167. True statement regarding surviving sepsis guideline:
b. Associated with H. pylori infection a. Activated protein C is useful
c. More common in male than female b. qSOFA should be assessed in case of prolong stay of patient
d. Better prognosis than diffuse in ICU
e. More common than diffuse type c. Suggest against the use of IV immunoglobulins in patients
160. Correct statement about Intussusception in children is/are with sepsis
all except: d. Recommend the use of erythropoietin for treatment of
a. Appendix is the most common leading point anemia associated with sepsis
b. Ileocolic is the most common site e. Recommend empiric broad-spectrum therapy
c. Usually presents with pain without vomiting
d. Can be corrected by air and barium enema
e. Diagnosis is confirmed on abdominal ultrasound
MCQs

Answer Key
154.
a, b, c. 155. a, b, c, d. 156. b, c, d. 157. e. 158. b, c, d, e. 159. a, b, c, d. 160. a, c.
161. a, c, d, e. 162. a, b. 163. b. 164. b. 165. a, b, c, d. 166. b, c. 167. b, c, e.

145
PGI Chandigarh Self-Assessment & Review: 2017–2013

Medicine •• A harsh ejection systolic murmur radiates to the neck, with


a soft second heart sound, particularly in those with calcific
valves.
122. Ans:  
(c) No increase in cardiac output despite exercise,
(d) Ejection systolic murmur radiating to neck may Clinical Features of Aortic Stenosis
present, (e) Left ventricular hypertrophy –– Davidson 22nd/620

[Ref: Harrison 19th/1529-33; Davidson 22nd/620-22, 616-17] Symptoms


•• Mild or moderate stenosis:usually asymptomatic
“The average time to death after the onset of various symptoms •• Exertional dyspnoea
is as follows: angina pectoris, 3 years; syncope, 3 years; dyspnea, •• Angina
2 years; congestive heart failure, 1.5–2 years. Moreover, in •• Exertional syncope
>80% of patients who died with AS, symptoms had existed for •• Sudden death
<4 years”- Harrison 19th/1532 •• Episodes of acute pulmonary oedema
“The murmur of AS is characteristically an ejection (mid)
systolic murmur that commences shortly after the S1, increases Signs
in intensity to reach a peak toward the middle of ejection, and •• Ejection systolic murmur
ends just before aortic valve closure. It is characteristically low- •• Slow-rising carotid pulse
pitched, rough and rasping in character, and loudest at the base •• Thrusting apex beat (LVpressure overload)

e
of the heart, most commonly in the second right intercostal •• Narrow pulse pressure

8/
space. It is transmitted upward along the carotid arteries”- •• Signs of pulmonary venous congestion (e.g. crepitations)
Harrison 19th/1531
123. Ans:  (a) Usually occurs after 2–3 month of acute exposure,
“Angina pectoris usually develops somewhat later and reflects

h,
(b) Involves both sensory and motor nerve,
an imbalance between the augmented myocardial oxygen
(d) Incomplete recovery
requirements and reduced oxygen availability. CAD may or ar
may not be present, although its coexistence is common among [Ref: Harrison 19th/2688; Davidson 22nd/222]
AS patients older than age 65”- Harrison 19th/1530-31
Organophosphate-induced Delayed Polyneuropathy
ig
Aortic Stenosis
–– Davidson 22nd/222
–– Davidson 22nd/620-22 •• Organophosphate-induced delayed polyneuropathy
nd

•• Cardiac output is initially maintained at the cost of a (OPIDN) is a rare complication that usually occurs 2–3
steadily increasing pressure gradient across the aortic valve. weeks after acute exposure. It is a mixed sensory/motor
The LV becomes increasingly hypertrophied and coronary
ha

polyneuropathy, especially affecting long myelinated neu-


blood flow may then be inadequate; patientsmay therefore rons, and appears to result from inhibition of enzymes other
develop angina, even in the absence of concomitant than AChE.
coronary disease.
IC

•• It is a feature of poisoning with some OPs such as


•• The fixed outflow obstruction limits the increase in trichlorocresylphosphate, but is less common with nerve
cardiac output required on exercise. Eventually, the LV agents.
can no longer overcome the outflow tract obstruction and •• Early clinical features are muscle cramps followed by
PG

pulmonary oedema supervenes. numbness and paraesthesiae, proceeding to flaccid


•• In contrast to patients with mitral stenosis, which tends paralysis of the lower and subsequently the upper limbs.
to progress very slowly, those with aortic stenosis typically Paralysis of the lower limbs is associated with foot drop and
remain asymptomatic for many years but deteriorate a high-stepping gait, progressing to paraplegia.Paralysis of
rapidly when symptoms develop, and death usually ensues the arms leads to wrist drop.
within 3–5 years of these. •• Sensory loss may also be present but is variable. Initially,
•• Aortic stenosis is commonly picked up in asymptomatic tendon reflexes are reduced or lost but mild spasticity may
patients at routine clinical examination but the three develop later.
cardinal symptoms are angina, breathlessness and syncope. •• There is no specific therapy for OPIDN. Regular physi-
•• Angina arises because of the increased demands of the otherapy may limit deformity caused by muscle wasting.
hypertrophied LV working against the high-pressure •• Recovery is often incomplete and may be limited to the
outflow tract obstruction, leading to a mismatch between hands and feet, although substantial functional recovery
oxygen demand and supply, but may also be due to after 1–2 years may occur, especially in younger patients.
coexisting coronary artery disease, especially in old age, “Organophosphate induced delayed polyneuropathy
when it affects over 50% of patients. (OPIDP) is a rare, delayed neurotoxic effect, which occurs
•• Exertional breathlessness suggests cardiac decompensation 1-5 weeks after severe toxicity from some cholinesterase
as a consequence of the excessive pressure overload placed inhibitors. No specific treatment has been identified. The early
Answers on the LV. administration of pralidoxime and atropine does not seem to
& •• Syncope usually occurs on exertion when cardiac output prevent the condition. Patients with mild cases recover over
Explanations
fails to rise to meet demand, leading to a fall in BP.

236
November  | 2016

several months; those with more serious polyneuropathies have persistent effects. Recovery affects only sensory nerves, while motor
neurons may permanently lose function”-www.atsdr.cdc.gov
Table (Harrison 19th/2688): Toxic neuropathies (partial list)

Drug Mechanism of Clinical Features Nerve Histopathology EMG/NCS


Neurotoxicity
Organophosphates Bind and inhibit Early features are those of Axonal degeneration Early: repetitive firing of
neuropathy target neuromuscular blockade with along with degeneration CMAPs and decrement with
esterase generalized weakness; later axonal of gracile fasciculus and repetitive nerve stimulation;
sensorimotor PN ensues corticospinal tracts late: axonal sensorimotor PN

124. Ans:  (b) Prothrombin time expressed as international concentration (<130 mEq/L [130 mmol/L]), an elevated
normalized.., (c) Serum creatinine estimation hepatic venous pressure gradient, persistent ascites, and
a low health-related quality of life appear to be additional
[Ref: Harrison 19th/1995-96,2069; CMDT 2016/693-94;Davidson
22nd/944]
independent predictors of mortality, and modifications of
the MELD score, including one that incorporates the serum

e
Model for End-Stage Liver Disease (MELD) System sodium (MELDNa), are under consideration.

8/
–– Harrison 19th/1995
•• Recently, the Child-Pugh system has been replaced by Table ( Davidson 22nd/944): One-year survival rate depending on
MELD system for the liver transplantation. The MELD MELD score

h,
score is a prospectively derived system designed to predict
the prognosis of patients with liver disease and portal MELD score One-year survival (%)
hypertension.
•• This score is calculated from three noninvasive variables:
ar No complications Complications
<9 97 90
the prothrombin time expressed as the international
ig
10–19 90 85
normalized ratio (INR), the serum bilirubin level, and the
20–29 70 65
serum creatinine concentration.
nd

•• The MELD system provides a more objective means of 30–39 70 50


assessing disease severity and has less center-to-center MELD From SI units
variation than the Child- Pugh score as well as a wider 10× (0.378 [In serum bilirubin (µMol/L) + 1.12 [In INR] + 0.957
ha

range of values. [In serum creatinine (mmol/L)] + 0.643)


•• MELD is currently used to establish priority listing for
liver transplantation. A similar system, PELD (pediatric Table ( CMDT 2016/693): Child-turcotte-pugh scoring system and
IC

end-stage liver disease), is based on bilirubin, INR, serum MELD for staging cirrhosis
albumin, age, and nutritional status and is used for children
<12 years of age. Child-Turcotte-Pugh scoring system
PG

“The Child–Pugh score and, more recently, the MELD Numerical Score
(Model for End-stage Liver Disease) score can be used to Parameter 1 2 3
assess prognosis. The MELD is more difficult to calculate at
Ascites None Slight Moderate to severe
the bedside but, unlike the Child–Pugh score, includes renal
function; if this is impaired, it is known to be a poor prognostic Encephalopathy None Slight to Moderate to severe
feature in end-stage disease . Although these scores give a guide moderate
to prognosis, the course of cirrhosis can be unpredictable, as Bilirubin (mg/dL) < 2.0 2–3 > 3.0
complications such as variceal bleeding may occur”- Davidson Albumin (g/dL) > 3.5 2.8–3.5 < 2.8
22nd/944 Prothrombin 1–3 4–6 > 6.0
Prognosis of Cirrhosis time (seconds
–– CMDT 2016/693-94 increased)
•• The consistency of the MELD score among different Total numerical score and corresponding
hospitals may be improved when the INR is calibrated Child-Turcotte-Pugh class
based on prothrombin time control samples that include
Score Class
patients with liver disease rather than those taking oral
anticoagulants. 5–6 A
•• A MELD score of >14 is required for liver transplant listing. 7–9 B
In patients with a relatively low MELD score (<21) and a 10–15 C
Answers
&
low priority for liver transplantation, a low serum sodium Explanations

237
PGI Chandigarh Self-Assessment & Review: 2017–2013

125. Ans:  All (a, b, c, d, e)


[Ref: Harrison 19th/803; CMDT 2016/273; Davidson 22nd/146]
Health care–associated pneumonia (HCAP) occurs in community members whose extensive contact with healthcare has changed their
risk for virulent and drug-resistant organisms”- CMDT 2016/272
Hospital-acquired pneumonia (HAP) occurs more than 48 hours after admission to the hospital or other health care facility and
excludes any infection present at the time of admission”- CMDT 2016/272
Table (CMDT 2016/273): Risk factors for health care–associated pneumonia

• Antibiotic therapy in the preceding 90 days.


• Acute care hospitalization for at least 2 days in the preceding 90 days
• Residence in a nursing home or extended care facility
• Home infusion therapy, including chemotherapy, within the past 30 days
• Long-term dialysis within the past 30 days
• Home wound care
• Family member with an infection involving a multiple drug-resistant pathogen
Immunosuppressive disease or immunosuppressive therapy

e

8/
Table (Harrison 19th/803): Clinical conditions associated with and likely pathogens in health care–associated pneumonia

Pathogen

h,
Condition MRSA Pseudomonas aeruginosa Acinetobacter spp. MDR Enterobacteriaceae
Hospitalization for ≥48 h √ √ ar √ √
Hospitalization for ≥2 days in prior 3 months √ √ √ √
ig
Nursing home or extended-care-facility residence √ √ √ √
Antibiotic therapy in preceding 3 months √ √
nd

Chronic dialysis √
Home infusion therapy √
ha

Home wound care √


IC

Family member with MDR infection √ √

Abbreviations: MDR, multidrug-resistant; MRSA, methicillin-resistant Staphylococcus aureus.


“Healthcare-associated pneumonia (HCAP) is a category of nosocomial pneumonia defined by the 2005 American Thoracic Society/
PG

Infectious Diseases Society of America (ATS/IDSA) guidelines to include any patient who has been hospitalized in an acute care hospital
for 2 or more days within the past 90 days; residents of a nursing home or long-term care facility; recipients of recent intravenous
antibiotic therapy, chemotherapy, or wound care within the past 30 days; or patients who have attended a hospital or hemodialysis
clinic. In creating this relatively new category the ATS/IDSA acknowledged that these patients are at increased risk for infection with
antibiotic-resistant organisms and that initial inadequate antibiotic coverage leads to increased mortality” www.ncbi.nlm.nih.gov

126. Ans:  (d) Schauman and asteroid bodies are pathognomic


[Ref: Davidson 22nd/709-11;Harrison 19th/ 2205-12; Robbins(SAE) 9th/693-94 ;CMDT 2016/292-93]

Sarcoidosis
“Hypercalcemia and/or hypercalciuria occurs in about 10% of sarcoidosis patients.The mechanism of abnormal calcium metabolism is
increased production of 1,25-dihydroxyvitamin D by the granuloma itself.”- Harrison 19th/ 2208
•• “Bronchoalveolar lavage fluid in sarcoidosis is usually characterized by an increase in lymphocyte and a high CD4/ CD8 ratio”
(CMDT 2016/293)
•• “The use of the lymphocyte markers CD4 and CD8 can be used to determine the CD4/CD8 ratio of these increased lymphocytes
in the BAL fluid. A ratio of > 3.5 is strongly supportive of sarcoidosis but is less sensitive than an increase in lymphocytes alone”
(Harrison 19th/2210)
•• “Sarcoidosis: Intra-alveolar and interstitial accumulation of CD4+ T cells, resulting in CD4: CD8 ratios ranging from 5:1 to 15:1”
Answers
&
(Robbins 9th/693)
Explanations

238
November  | 2016

•• “Serum levels of angiotensin-converting enzyme (ACE) Exudative Pleural Effusions


can be helpful in the diagnosis of sarcoidosis. However, the
test has somewhat low sensitivity and specificity. Elevated • Collagen vascular diseases • Trapped lung
ƒƒ Rheumatoid pleuritis • Radiation therapy
levels of ACE are reported in 60% of patients with acute
disease and only 20% of patients with chronic disease. ƒƒ Systemic lupus • Post-cardiac injury syndrome
erythematosus • Hemothorax
Although there are several causes for mild elevation of
ƒƒ Drug-induced lupus • Iatrogenic injury
ACE, including diabetes, elevations of >50% of the upper
ƒƒ Immunoblastic • Ovarian hyperstimulation
limit of normal are seen in only a few conditions including
lymphadenopathy syndrome
sarcoidosis, leprosy, Gaucher’s disease, hyperthyroidism, ƒƒ Sjögren’s syndrome • Pericardial disease
and disseminated granulomatous infections such as miliary ƒƒ Granulomatosis with • Chylothorax
tuberculosis.”- Harrison 19th/ 2209 polyangiitis (Wegener’s)
•• “Schauman and asteroid bodies :Although characteristic, ƒƒ Churg-Strauss syndrome
these cells are not pathognomic of sarcoidosis because they
may be encountered in other granulomatous diseases (e.g., “The post-cardiac injury (Dressler’s) syndrome (PCIS) can occur
tuberculosis)” (Robbins 9th/693) after any type of cardiac injury and is thought to be due to anti-
•• The granuloma is the pathologic hallmark of sarcoidosis myocardial antibodies. Pleural effusions are common with PCIS.
•• A positive gallium scan can support the diagnosis if Following CABG surgery, most patients will have a small unilateral
left-sided pleural effusion, and approximately 10% of patients will

e
increased activity is noted in the parotids and lacrimal
glands (Panda sign) or in the right paratracheal and left have a larger effusion. These large effusions can be separated into

8/
hilar area (lambda sign) early effusions occurring within the first 30 days of surgery that
•• The Kviem-Siltzbach procedure is a specific diagnostic test are bloody exudates with a high percentage of eosinophils, and late
for sarcoidosis effusions occurring more than 30 days after surgery that are clear

h,
yellow lymphocytic exudates”- www.ncbi.nlm.nih.gov
127. Ans:  (a) Urinothorax, (c) Nephrotic syndrome, ar
(d) Myxedema, (e) Congestive heart failure 128. Ans:  (b) Associated with inflammatory bowel disease,
(d) Periductal fibrosis of smaller bile ducts
[Ref: Harrison 19th/1718; CMDT 2016/310; Davidson 22nd/661-62]
ig
[Ref: Davidson 22nd/965-66;Robbins 9th/858-60]
Post-cardiac injury (Dressler’s) syndrome (PCIS) causes exu-
dative pleural effusion Cirrhosis due to primary sclerosing cholangitis: Following
nd

changes are seen-fibrosing cholangitis, periductal fibrosis,


Table ( Harrison 19th/1718): Differential diagnoses of pleural dilation of intervening bile ducts and cholestatis with full blown
effusions picture of biliary cirrhosis”-Harshmohan 7th/611
ha

Primary sclerosing cholangitis causes micronodular cirrhosis-


Transudative Pleural Effusions
Harshmohan 7th/611; www.cirrhosisoftheliver.co
• Congestive heart failure • Peritoneal dialysis
IC

• Cirrhosis • Superior vena cava


• Pulmonary embolization obstruction(!! It is removed in Primary Sclerosing Cholangitis
• Nephrotic syndrome 19th edition) –– Robbins 9th/859-60
• Myxedema
PG

•• PSC is characterized by inflammation and obliterative


• Urinothorax
fibrosis of intrahepatic and extrahepatic bile ducts with
Exudative Pleural Effusions dilation of preserved segments.
• Neoplastic diseases • Post-coronary artery bypass •• Characteristic “beading” on radiographs of the intrahepatic
ƒƒ Metastatic disease surgery and extrahepatic biliary tree is attributable to these irregular,
ƒƒ Mesothelioma • Asbestos exposure biliary strictures and dilations.
• Infectious diseases • Sarcoidosis •• Inflammatory bowel disease , particularly ulcerative colitis,
ƒƒ Bacterial infections • Uremia coexists in approximately 70% of individuals with PSC
ƒƒ Tuberculosis • Meigs’ syndrome •• Morphologic changes differ between the large ducts (in-
ƒƒ Fungal infections • Yellow nail syndrome
trahepatic and extrahepatic) and the smaller intrahepatic
ƒƒ Viral infections • Drug-induced pleural disease
ƒƒ Parasitic infections ƒƒ Nitrofurantoin
ducts.
• Pulmonary embolization ƒƒ Dantrolene
•• The smaller ducts, often have little inflammation and show
• Gastrointestinal disease ƒƒ Methysergide a striking circumferential “onion skin” fibrosis around
ƒƒ Esophageal perforation ƒƒ Bromocriptine an increasingly atrophic duct lumen , eventually leading to
ƒƒ Pancreatic disease ƒƒ Procarbazine obliteration by a “tombstone” scar.
ƒƒ Intraabdominal abscesses ƒƒ Amiodarone •• As the disease progresses the liver becomes markedly
ƒƒ Diaphragmatic hernia ƒƒ Dasatinib cholestatic, culminating in biliary cirrhosis much like that
ƒƒ After abdominal surgery seen with chronic obstruction and primary biliary cirrhosis.
ƒƒ Endoscopic variceal
•• Biliary intraepithelial neoplasia may develop and Answers
sclerotherapy &
cholangiocarcinoma appears usually with a fatal outcomes. Explanations
ƒƒ After liver transplant
Contd...

239
PGI Chandigarh Self-Assessment & Review: 2017–2013

Primary Sclerosing Cholangitis (PSC) •• There is a characteristic clinical triad resulting from these
–– Davidson 22nd/965-66 pancreatic lesions: (1) Attacks of hypoglycemia occur
•• Primary sclerosing cholangitis (PSC) is a cholestatic liver with blood glucose levels below 50 mg/dL of serum; (2)
disease caused by diffuse inflammation and fibrosis; it the attacks consist principally of such central nervous
can involve the entire biliary tree and leads to the gradual system manifestations as confusion, stupor, and loss of
obliteration of intrahepatic and extrahepatic bile ducts, consciousness; and (3) the attacks are precipitated by
and ultimately biliary cirrhosis, portal hypertension and fasting or exercise and are promptly relieved by feeding or
hepatic failure. parenteral administration of glucose”
•• The cause of PSC is unknown but there is a close association •• Insulinomas are most often found within the pancreas and
with inflammatory bowel disease, particularly ulcerative are generally benign.
colitis. About two-thirds of patients have coexisting ulcera- •• Most are solitary, although multiple tumors may be en-
tive colitis, and PSC is the most common form of chronic countered.
liver disease in ulcerative colitis. •• Solitary tumors are usually small (often < 2 cm in diameter),
•• PSC is twice as common in young men. Most patients encapsulated, pale to red-brown nodules located anywhere
present at age 25–40 years, although the condition may be in the pancreas.
diagnosed at any age and is an important cause of chronic •• Histologically, these benign tumors look remarkably
liver disease in children. like giant islets, with preservation of the regular cords of

e
monotonous cells and their orientation to the vasculature.
Diseases Associated with Primary Sclerosing Cholangitis

8/
Not even the malignant lesions present much evidence
–– Davidson 22nd/965 of anaplasia, and they may be deceptively encapsulated.
•• Ulcerative colitis Deposition of amyloid is a characteristic feature of many
•• Crohn’s colitis

h,
insulinomas
•• Chronic pancreatitis
•• Retroperitoneal fibrosis Insulinomas
•• Riedel’s thyroiditis
ar
–– Harrison 19th/569
•• Retro-orbital tumours
•• Insulinomas are generally small (>90% are <2 cm) and
ig
•• Immune deficiency states
•• Sjögren’s syndrome usually not multiple (90%); only 5–15% are malignant,
and they almost invariably occur only in the pancreas,
nd

•• Angio-immunoplastic lymphadenopathy
•• Histiocytosis X distributed equally in the pancreatic head, body, and tail.
•• Autoimmune haemolytic anaemia •• Insulinomas should be suspected in all patients with
ha

•• Autoimmune pancreatitis/ IgG4-associated cholangitis hypoglycemia, especially when there is a history suggesting
that attacks are provoked by fasting, or with a family history
129. Ans:  (a) Encapsulated, (c) Associated with MEN-I, of MEN 1.
IC

(d) Enucleation is the treatment…, (e) Histology •• Only 5–15% of insulinomas are malignant; therefore, after
similar to normal β-cells appropriate imaging , surgery should be performed. In
different studies, 75–100% of patients are cured by surgery.
[Ref: Harrison 19th/569;Robbins 9th/1121; L and B 26th/787-89]
PG

“Benign insulinoma: Tumour enucleation is the technique of 130. Ans:  (b) Increased CD8+ T cells in…, (c) Manifests mainly
choice”-L and B 26th/789 as an…, (d) For severe acute cases…, (e) Interstitial
“Grossly insulinoma is usually solitary and well-encapsulated inflammatory…
tumour which may vary in size from 0.5 to 10 cm. Rarely, they
[Ref: Robbins 9th/694-95;Davidson 22nd/719-20; Harrison 19th/ 1681-
are multiple. Microscopically, the tumour is composed of cords
83;CMDT 2016/306-07]
and sheet of well-differentiated β-cells which do not differ from
normal cells”-Harshmohan 7th/818 Hypersensitivity Pneumonitis
“Insulinoma: About 90% are solitary and about 10% are –– Robbins 9th/694-95
multiple and are associated with MEN1. About 10% are •• Most commonly, hypersensitivity results from the inhala-
malignant”-L and B 26th/788 tion of organic dust containing antigens made up of the
“Majority of the insulinomas occur in the tail and body of spores of thermophilic bacteria, fungi, animal proteins, or
pancreas and majority of them are benign, one third are bacterial products.
malignant and one-third are multiple”-Manipal manual of •• Bronchoalveolar lavage specimens also consistently
Surgery 3rd/501 demonstrate increased numbers of both CD4+ and CD8+
Insulinomas T lymphocytes.
–– Robbins 9th/1121 •• The clinical manifestations are varied. Acute attacks, which
•• “β-cell tumors (insulinomas) are the most common of follow inhalation of antigenic dust in sensitized patients,
Answers pancreatic endocrine neoplasms and may be responsible consist of recurring episodes of fever, dyspnea, cough, and
&
Explanations for the elaboration of sufficient insulin to induce clinically leukocytosis.
significant hypoglycemia.

240
November  | 2016

Hypersensitivity Pneumonitis •• “Middle-aged women (female-to-male ratio, 9:1) are


–– CMDT 2016/306-07 primarily affected, although Sjögren’s syndrome may occur
•• It is manifested mainly as an occupational disease, in at any age, including childhood”-Harrison
which exposure to inhaled organic agents leads to acute •• The principal oral symptom of Sjögren’s syndrome is dry-
and eventually chronic pulmonary disease ness (xerostomia). Patients report difficulty in swallowing
•• Treatment of hypersensitivity pneumonitis consists of dry food, an inability to speak continuously, a burning sen-
identification of the offending agent, avoidance of further sation, an increase in dental caries, and problems in wearing
exposure and in severe acute or protracted cases, oral complete dentures
corticosteroids (prednisone, 0.5 mg/kg daily as a single •• Enlargement of the parotid or other major salivary glands
occurs in two-thirds of patients with primary Sjögren’s
morning dose for 2 weeks, tapered to nil over 4–6 weeks)
syndrome but is uncommon in those with the secondary
may be given
syndrome.
Hypersensitivity Pneumonitis (HP) •• Sjögren’s syndrome causes bilateral parotid enlargement-
–– Davidson 22nd/719-20 Harrison 19th/2167(table)
•• BAL fluid usually shows an increase in the number of CD8+ •• “Occasionally, there is enlargement of the salivary glands,
T lymphocytes more commonly the parotid rather than the submandibular
•• HRCT showing typical patchy ground glass opacification. glands.The glands are occasionally painful, and the patient

e
•• Histology shows evidence of an interstitial inflammatory sometimes develops a bacterial sialadenitis due to ascending
infiltrate in the lung, expanding alveolar walls, with a infection from the associated xerostomia”-L and B 26th/737

8/
peribronchial distribution. •• Lymphoma is a well-known manifestation of Sjögren’s
•• In acute cases, prednisolone should be given for 3–4 weeks, syndrome that usually presents later in the illness.

h,
starting with an oral dose of 40 mg per day. Persistent parotid gland enlargement, purpura, leukopenia,
Hypersensitivity Pneumonitis (HP) cryoglobulinemia, low C4 complement levels, and ectopic
germinal centers in minor salivary gland biopsy samples are
–– Harrison 19th/ 1681-83
ar manifestations suggesting the development of lymphoma
•• The incidence and prevalence of HP are variable, depending
ig
on geography, occupation, avocation, and environment
•• The various antigens and environmental conditions Table (Davidson 22nd /1114): Features of sjogren’s syndrome
nd

associated with HP give rise to an expansive list of monikers


given to specific forms of HP. Risk Factors
•• Most cases of HP have a CD4+/CD8+ lymphocyte ratio of • Age of onset 40-60 • HLA-B8/DR3
ha

less than 1, but again, this is not a specific finding and has • Female > male
limited utility in the diagnosis of HP(Note: Normal CD4+/
Common Clinical Features
CD8+ ratio is 2)
IC

•• Within the alveolar spaces and in the interstitium, a mixed • Keratoconjunctivitis sicca • Non-erosive arthritis
cellular infiltrate with a lymphocytic predominance is • Xerostomia • Raynaud’s phenomenon
observed that is frequently patchy in distribution. • Salivary gland enlargement • Fatigue
PG

•• Although glucocorticoids do not change the long-term


Less Common Features
outcome in these patients, they can accelerate the resolution
of symptoms. While there is significant variability in the • Low-Grade Fever • Peripheral neuropathy
approach to glucocorticoid therapy by individual clinicians, • Interstitial Lung Disease • Lymphadenopathy
prednisone therapy can be initiated at 0.5–1 mg/kg of ideal • Anaemia, Leucopenia • Lymphoreticular lymphoma
body weight per day (not to exceed 60 mg/d or alternative • Thrombocytopenia • Glomerulonephritis
• Cryoglobulinaemia • Renal tubular acidosis
glucocorticoid equivalent) over a duration of 1–2 weeks,
• Vasculitis
followed by a taper over the next 2–6 weeks.
Autoantibodies Frequently Detected
131. Ans:  (e) Males are affected more than females
• RF • SS-B (anti-La)
[Ref: Harrison 19th/2166-68; CMDT 2016/836-37; Davidson 22nd/1114; • ANA • Gastric parietal cell
Robbins 9th/226-27; L and B 26th/737] • SS-A (anti-Ro) • Thyroid
Sjögren’s Syndrome Associated Autoimmune Disorders
–– Harrison 19th/2166-67
• SLE • Primary biliary cirrhosis
•• “Females are affected more than males in the ratio 10:1”- L
• Progressive systemic sclerosis • Chronic active hepatitis
and B 26th/737 • Myasthenia gravis

Answers
&
Explanations

241
PGI Chandigarh Self-Assessment & Review: 2017–2013

132. Ans:  (a) Seropositive for…, (b) Splenomegaly, (c) Long HIV-Related Malignancies
standing… (e) Keratoconjunctivitis sicca –– CMDT 2016/1322
–– [Ref: Robbins 9th/606 ; CMT 2016/821; Harrison 19th/418,2138 ; •• Four cancers are currently included in the CDC classifica-
Davidson 22nd/1100] tion of AIDS: Kaposi sarcoma, non-Hodgkin lymphoma,
Felty’s Syndrome primary lymphoma of the brain, and invasive cervical car-
–– Harrison 19th/418 cinoma.
•• “Felty’s syndrome is defined by the clinical triad of •• Non-Hodgkin’s lymphoma: The best studied example is
neutropenia, splenomegaly, and nodular RA and is seen in Burkitt’s lymphoma”
less than 1% of patients, although its incidence appears to “The AIDS-defining cancers are Kaposi’s sarcoma, cervical
be declining in the face of more aggressive treatment of the cancer and non-Hodgkin lymphoma (NHL). NHL may occur
joint disease. It typically occurs in the late stages of severe at any CD4 count but is more commonly seen below 200 cells/
RA”-Harrison 19th/2138 mm3. Almost all NHL are B-cell tumours and most are stage 4
•• Spleen-produced antibodies can shorten neutrophil life when the patient presents. The incidence of a number of other
span, while large granular lymphocytes can attack marrow cancers induced by viruses is also increased in HIV-infected
neutrophil precursors. people”- Davidson 22nd/405
•• Splenectomy may increase the neutrophil count in Felty’s

e
syndrome and lower serum neutrophil binding IgG. Table ( Davidson 22nd/405): Approximate incidence ratio of virus-

8/
•• Some Felty’s syndrome patients also have neutropenia
related cancers in HIV patients compared to the general
associated with an increased number of LGLs. population
Table (davidson 22nd/1100): Felty’s syndrome

h,
Risk factors arViral cancers Incidence ratio

• Age of onset 50-70 yrs • Deforming but inactive disease Human herpesvirus 8-related
• Female > male • Seropositive for RF Kaposi’s sarcoma 3600
ig
• Caucasians> blacks
• Long-standing RA Epstein–Barr virus-related
nd

Common clinical features Non-Hodgkin lymphoma 80


Hodgkin lymphoma 10
• Splenomegaly • Keratoconjunctivitis sicca
Lymphadenopathy Vasculitis, leg ulcers
ha

• •
Human papillomavirus-related
• Weight loss • Recurrent infections
Cervical cancer 6
• Skin pigmentation • Nodules
Vulval cancer 6
IC

Laboratory findings Anal cancer 30


• Normochromic, • Thrombocytopenia Penile cancer 4
normocytic anaemia • Impaired T-and B-cell immunity
Hepatitis B/C virus-related
PG

• Neutropenia
• Abnormal liver function Hepatoma 5

133. Ans:  (a) Primary CNS lymphoma, (b) Cervical cancer,


(c) Kaposi sarcoma, (d) Ovarian cancer 134. Ans:  (a) Breast cancer, (c) Non-small lung cancer,
(d) Prostate cancer, (e) Multiple myeloma
[Ref: Davidson 22nd/405; CMDT 2016/1322; Harrison 19th/1268; Da-
vidson 22nd/405 ; Ananthanarayan 9th/576-77; Lippincott Microbiology [Ref: Davidson 22nd/271; Williams Textbook of Endocrinology, 11th ed;
3rd/ 302] Harrison 19th/608-09; CMDT 2016/ 1586-88; Robbins 9th/330- 31]

Neoplastic Diseases “Hypercalcemia is probably the most common paraneoplastic


syndrome; overtly symptomatic hypercalcemia is most
–– Harrison 19th/1268
often related to some form of cancer rather than to
•• The neoplastic diseases considered to be AIDS defining
hyperparathyroidism. Two general processes are involved
conditions are Kaposi’s sarcoma, non-Hodgkin’s lymphoma,
in cancer-associated hypercalcemia: osteolysis induced by
and invasive cervical carcinoma.
cancer, whether primary in bone, such as multiple myeloma,
•• In addition, there is also an increase in the incidence of a va-
or metastatic to bone from any primary lesion, and, the
riety of non-AIDS-defining malignancies including Hodg- production of calcemic humoral substances by extraosseous
kin’s disease; multiple myeloma; leukemia; melanoma;and neoplasms. Hypercalcemia owing to skeletal metastases is not a
cervical, brain, testicular, oral, lung, gastric, liver, renal, paraneoplastic syndrome” (Robbins 9th/330)
Answers and anal cancers.
&
Explanations

242
November  | 2016

Table ( Harrison 19th/609): Paraneoplastic syndromes caused by 135. Ans:  (a) Adenosine deaminase…, (b) Haematopoietic
ectopic hormone production stem…, (c) Most common inheritance…,
(e) Increased risk of infection…
Paraneoplastic Ectopic hormone Typical tumor types
Syndrome [Ref: Robbins 9th/239- 40; Harrison 19th/2107-08; Ananthanarayan 9th/
174-75]
Hypercalcemia Parathyroid hormone- Squamous cell (head
of malignancy related protein (PTHrP) and neck, lung, skin),
Severe Combined Immunodeficiency (SCID)
breast, genitourinary,
gastrointestinal –– Harrison 19th/2107-08
•• SCID is characterized by a profound block in T cell
1, 25 dihydroxyvitamin Lymphomas
D development and thus the complete absence of these cells.
•• The most frequent clinical manifestations are recurrent oral
Parathyroid hormone Lung, ovary candidiasis, failure to thrive, and protracted diarrhea and/or
(rare)
acute interstitial pneumonitis caused by Pneumocystis jiroveci
Prostaglandin E2 Renal, lung •• Diagnosis of SCID can be suspected based on the patient’s
(PGE2) (rare) clinical history and, possibly, a family history of deaths
“Lung carcinoma, breast carcinoma, and multiple myeloma in very young children (suggestive of either X-linked or

e
account for more than 50% of all cases of malignancy-associated recessive inheritance).
hypercalcemia. Lung carcinomas that produce hypercalcemia •• Lymphocytopenia is strongly suggestive of SCID in more

8/
have squamous or large cell histology, whereas small cell than 90% of cases.
carcinoma almost never causes hypercalcemia. Among other •• The absence of a thymic shadow on a chest x-ray can also be
suggestive of SCID. An accurate diagnosis relies on precise

h,
solid tumors, the most common are squamous and renal
carcinomas. Gastrointestinal tumors and prostate carcinoma determination of the number of circulatingT, B, and NK
are less common causes of hypercalcemia”- Williams Textbook lymphocytes and their subsets.
of Endocrinology, 11thed
ar •• Although counts are usually low (<500/μL of blood), higher
maternal T cell counts may, under some circumstances,
ig
Table: (Williams Textbook of Endocrinology11th ed.): Malignancy- initially mask the presence of SCID.
associated hypercalcemia •• Severe combined immunodeficiency caused by a
nd

cytokine-signaling deficiency: The most frequent SCID


Primary Site No. (%) of Cases
phenotype (accounting for 40–50% of all cases) is the
Lung 111 (25.0) absence of both T and NK cells
ha

Breast 87 (19.6) •• Purine metabolism deficiency: Ten to 20% of SCID


Multiple myeloma 43 (9.7) patients exhibit a deficiency in adenosine deaminase
(ADA), an enzyme of purine metabolism that deaminates
IC

Head and neck 36 (8.1)


adenosine (ado) and deoxyadenosine (dAdo).
Renal and urinary tract 35 (7.9)
•• Patients with SCID require appropriate care with aggressive
Esophagus 25 (5.6) antiinfective therapies, immunoglobulin replacement, and
PG

Female genitalia 24 (5.2) (when necessary) parenteral nutrition support. In most


Unknown primary 23 (5.2) cases, curative treatment relies on HSCT. Today, HSCT
Lymphoma 14 (3.2) provides a very high curative potential for SCID patients
who are otherwise in reasonably good condition.
Colon 8 (1.8)
•• Gene therapy has been found to be successful for cases of
Liver, biliary 7 (1.6) X-linked SCID (γc deficiency) and SCID caused by an ADA
Skin 6 (1.4) deficiency,although toxicity has become an issue in the
Other 25 (5.6) treatment of the former disease that may now be overcome
by use of newly generated vectors.
Table (Robbins 9th/331): Paraneoplastic syndromes •• Lastly, a third option for the treatment of ADA deficiency
consists of enzyme substitution with a pegylated enzyme.
Clinical Major Forms of Causal
Syndromes Underlying Cancer Mechanism
Severe Combined Immunodeficiency
Hypercalcemia Squamous cell Parathyroid
carcinoma of lung hormone-related –– Ananthanarayan 9th/ 174-75
Breast carcinoma; Renal protein (PTHRP), •• They are inherited in the autosomal recessive mode
carcinoma TGF-α, TNF, IL-1 •• Adenosine deaminase deficiency: This the first immunodefi-
Adult T-cell leukemia/ ciency disease associated with an enzyme deficiency
lymphoma •• Deficiency of both humoral and CMI response Answers
&
Explanations

243
PGI Chandigarh Self-Assessment & Review: 2017–2013

Severe Combined Immunodeficiency Hyperkalaemic distal renal tubular acidosis


–– Robbins 9th/239- 40 • Hypoaldosteronism (primary or • Drugs
•• Persons with SCID are extremely susceptible to recurrent, secondary)  Amiloride
severe infections by a wide range of pathogens, including • Obstructive nephropathy  Spironolactone
Candida albicans, Pneumocystis jiroveci, Pseudomonas, • Renal transplant rejection
cytomegalovirus, varicella, and a whole host of bacteria.
•• Without HSC transplantation, death occurs within the first 137. Ans:  (a) Lactic acidosis, (b) Ethylene glycol…
year of life. (c) Aspirin overdose
•• The most common form, accounting for 50% to 60% of
[Ref: Harrison 19th/318; CMDT 2016/890; Davidson 22nd/445]
cases, is X-linked, and hence SCID is more common in boys
than in girls. The genetic defect in the X-linked form is a
Table (Harrison 19th/318): Causes of high anion–gap metabolic
mutation in the common γ-chain (γc) subunit of cytokine
acidosis
receptors
•• Autosomal recessive SCID: The remaining forms of SCID •• Lactic acidosis
are autosomal recessive disorders. The most common cause •• Ketoacidosis (diabetic, alcoholic, starvation)
of autosomal recessive SCID is a deficiency of the enzyme •• Toxins (ethylene glycol, methanol, starvation, propylene
adenosine deaminase (ADA). glycol, pyroglutamic acid)

e
•• Currently, HSC transplantation is the mainstay of treatment, •• Renal failure (acute and chronic)
but X-linked SCID is the first human disease in which gene

8/
therapy has been successful. Table (Davidson 22nd/445): Causes of metabolic acidosis

136. Ans:  (a) Fanconi anemia, (b) Multiple myeloma, Disorder Mechanism

h,
(c) Lead poisoning A. Normal anion gap
[Ref: Harrison 19th/320; Davidson 22nd/446] Ingestion or infusion of Therapeutic infusion of or poisoning
arinorganic acid with NH4Cl, HCl
Renal Tubular Acidosis
Gastrointestinal HCO3 loss Loss of HCO3 in diarrhoea, small
ig
–– Davidson 22nd/446
bowel fistula, urinary diversion
•• Renal tubular acidosis (RTA) should be suspected when
procedure
nd

there is a hyperchloraemic acidosis with a normal anion gap


and no evidence of gastrointestinal disturbance. Renal tubular acidosis (RTA) Urinary loss of HCO3 in proximal
RTA; impaired tubular acid secretion
•• The urine pH is inappropriately high (> 5.5) in the presence
in distal RTA
ha

of systemic acidosis.
B. Increased anion gap
•• RTA can be caused by a defect in one of three processes:
impaired bicarbonate reabsorption in the proximal tubule Endogenous acid load
IC

(proximal RTA); impaired acid secretion in the late distal Diabetic ketoacidosis Accumulation of ketones with
tubule or cortical collecting duct intercalated cells (classical hyperglycaemia
distal RTA); or impaired sodium reabsorption in the late
Starvation ketosis Accumulation of ketones without
PG

distal tubule or cortical collecting duct, which is associated hyperglycaemia


with reduced secretion of both potassium and H+ ions
(hyperkalaemic distal RTA). Lactic acidosis Shock, liver disease, drugs
Renal failure Accumulation of organic acids
Table (Davidson 22nd/446): Causes of renal tubular acidosis
Exogenous acid load
Proximal renal tubular acidosis Aspirin poisoning Accumulation of salicylate
• Inherited • Heavy metal toxicity Methanol poisoning Accumulation of formate
 Fanconi’s syndrome  Lead, cadmium and
 Cystinosis mercury poisoning Ethylene glycol poisoning Accumulation of glycolate, oxalate
 Wilson’s disease • Drugs
Table (CMDT2016/890): Anion gap in metabolic acidosis
• Paraproteinaemia  Carbonic anhydrase
 Myeloma  Inhibitors Increased (> 12 mEq)
• Amyloidosis  Ifosfamide
• Hyperparathyroidism A. Metabolic anion
Classical distal renal tubular acidosis Diabetic ketoacidosis
Inherited
• Hyperglobulinaemia

Alcoholic ketoacidosis
Autoimmune diseases
• Toxins and drugs

Answers  Systemic lupus  Toluene Lactic acidosis
&  Erythematosus  Lithium
Explanations
 Sjogren’s syndrome  Amphotericin
Chronic kidney disease (advanced stages)

244
November  | 2016

Increased (> 12 mEq) light chains. Causes include acute tubular necrosis, toxic
injury (lead, aminoglycosides), drug-induced interstitial
Starvation nephritis, and hereditary metabolic disorders (Wilson
disease and Fanconi syndrome).
Metabolic alkalosis (increased number of negative charges
on protein) Table: Causes of Proteinuria (Excluding glomerular cause) patient.
5–oxoproline acidosis from acetaminophen toxicity info/in/doctor

B. Drug or chemical anion Transient proteinuria


Salicylate intoxication • Emotional stress.
• Exercise.
Sodium carbenicillin therapy • Fever.
Methanol (formic acid) • Urinary tract infection.
• Orthostatic (postural) proteinuria.
Ethylene glycol (oxalic acid) • Seizures.
Normal (6–12 mEq) • Persistent proteinuria.

A. Loss of HCO3 Tubular causes

e
• Aminoaciduria.
Diarrhea • Drugs (eg, NSAIDs, antibiotics).

8/
B. Chloride retention • Fanconi’s syndrome.
• Heavy metal ingestion.
Renal tubular acidosis
Overflow causes

h,
138. Ans:  (b) Wilson disease, (c) Lead poisoning, (d) Fanconi • Haemoglobinuria.
syndrome
ar•

Multiple myeloma.
Myoglobinuria.
[Ref: Harrison 19th/293; CMDT 2016/898-99]
ig
Other important causes (likely to have multiple pathologies)
Proteinuria
• Pre-eclampsia/eclampsia.
–– CMDT 2016/898-99
nd

•• Proteinuria is defined as excessive protein excretion in the 139. Ans:  (b) Oat, rye and barley can be safely given, (e) No risk
urine, generally greater than 150–160 mg/24 h in adults. for development of cancer
•• Significant proteinuria is a sign of an underlying renal
ha

abnormality, usually glomerular in origin when greater than [Ref: Harrison 19th/1940-42;Davidson 22nd/880-82]
1 g/d. Celiac Disease
IC

•• There are four primary reasons for development of pro-


–– Harrison 19th/1940-42
teinuria:
•• “It is an inflammatory disorder of the small bowel occurring
 Functional proteinuria is a benign process stemming
in genetically susceptible individuals, which results from
from stressors such as acute illness, exercise, and “ortho-
PG

intolerance to wheat gluten and similar proteins found in


static proteinuria.”
rye, barley and, to a lesser extent, oats”- Davidson 22nd/880
 Overload proteinuria can result from overproduction
•• The symptoms range from significant malabsorption of
of circulating, filterable plasma proteins (monoclonal
multiple nutrients, with diarrhea, steatorrhea, weight loss,
gammopathies), such as Bence Jones proteins associated
and the consequences of nutrient depletion (i.e., anemia and
with multiple myeloma. Urinary protein electrophoresis
metabolic bone disease), to the absence of gastrointestinal
will exhibit a discrete protein peak. Other examples
symptoms despite evidence of the depletion of a single
of overload proteinuria include myoglobinuria in
nutrient (e.g., iron or folate deficiency, osteomalacia, edema
rhabdomyolysis and hemoglobinuria in hemolysis.
from protein loss).
 Glomerular proteinuria results from effacement of
•• One environmental factor is the clear association of the
epithelial cell foot processes and altered glomerular
disease with gliadin, a component of gluten that is present
permeability with an increased filtration fraction of
in wheat, barley, and rye.
normal plasma proteins. Glomerular diseases exhibit
•• An immunologic component in the pathogenesis of celiac
some degree of proteinuria. The urinary electrophoresis
disease is critical and involves both adaptive and innate
will have a pattern exhibiting a large albumin spike
immune responses. Serum antibodies—IgA antigliadin,
indicative of increased permeability of albumin across a
antiendomysial, and anti-tTG antibodies— are present,
damaged glomerular basement membrane (GBM).
but it is not known whether such antibodies are primary
 Tubular proteinuria occurs as a result of faulty
or secondary to the tissue damage. The presence of
reabsorption of normally filtered proteins in the proximal Answers
antiendomysial antibody is 90–95% sensitive and 90–95%
tubule, such as β2-microglobulin and immunoglobulin &
specific Explanations

245
PGI Chandigarh Self-Assessment & Review: 2017–2013

•• A small-intestinal biopsy is required to establish a diagnosis of celiac disease . A biopsy should be performed when patients have
symptoms and laboratory findings suggestive of nutrient malabsorption and/or deficiency as well as a positive tTG antibody test.
•• Gluten is ubiquitous, and a significant effort must be made to exclude all gluten from the diet. Use of rice flour in place of wheat flour
is very helpful, and several support groups provide important aid to patients with celiac disease and to their families. More than 90%
of patients who have the characteristic findings of celiac disease respond to complete dietary gluten restriction.
•• Celiac disease is associated with dermatitis herpetiformis (DH), but this association has not been explained
•• The most important complication of celiac disease is the development of cancer. The incidences of both gastrointestinal and
nongastrointestinal neoplasms as well as intestinal lymphoma are elevated among patients with celiac disease.

140. Ans:  (a) Myasthenia gravis, (b) Acute exacerbation in COPD, (c) Acute severe asthma
[Ref: Harrison 19th/1731-32;Davidson 22nd/663-65]

Type II Respiratory Failure


–– Harrison 19th/1732
•• This type of respiratory failure is a consequence of alveolar hypoventilation and results from the inability to eliminate carbon
dioxide effectively.
•• Mechanisms are categorized by impaired central nervous system (CNS) drive to breathe, impaired strength with failure of

e
neuromuscular function in the respiratory system, and increased load(s) on the respiratory system.

8/
•• Reasons for diminished CNS drive to breathe include drug overdose, brainstem injury, sleep-disordered breathing, and severe
hypothyroidism.
•• Reduced strength can be due to impaired neuromuscular transmission (e.g., myasthenia gravis,Guillain-Barre syndrome,

h,
amyotrophic lateral sclerosis) or respiratory muscle weakness (e.g., myopathy, electrolyte derangements, fatigue).
ar
Table (Davidson 22nd/664): How to interpret blood gas abnormalities in respiratory failure
ig
Type I Type II
Hypoxia (PaO2 <8.0kPa (60 mmHg) Hypoxia (PaO2 <8.0 kPa (60 mmHg)
nd

Normal or low PaCO2 (<6.6 kPa (50 mmHg) Raised PaCO2 (> 6.6 kPa (50 mmHg)
Acute Chronic Acute Chronic
ha

H+ → → ↑ → or ↑
Bicarbonate → → → ↑
Causes Acute asthma COPD Acute severe asthma COPD
IC

Pulmonary oedema Lung fibrosis Acute exacerbation of COPD Sleep apnoea


Pneumonia Lymphangitis Upper airway obstruction Kyphoscoliosis
Lobar collapse Carcinomatosa Acute neuropathies/paralysis Myopathies/muscular
PG

Pneumothorax Right-to-left shunts Narcotic drugs dystrophy


Pulmonary embolus Primary alveolar hypoventilation Ankylosing spondylitis
ARDS Flail chest injury

141. Ans:  (b) Acute intermittent porphyria


[Ref: Davidson 22nd/459, 1260; Harrison 19th/ 2521-30; CMDT 2016/1655-56]
“In contrast to other forms of porphyria, cutaneous photosensitivity is absent in AIP”- CMDT 2016/1655

Answers
&
Explanations

246
November  | 2016

e
8/
h,
ar
ig
Fig.:  (Davidson 22nd/459): Enzyme defects responsible for the porphyrias. (N = neurovisceral; P = photosensitive)
nd

142. Ans:  (a) Low pitch, (c) Absent in atrial…, (d) Produced in the ventricle…, (e) Present in severe left ventricular…
[Ref: Harrison 19th/1448;Davidson 22nd/560; Braunward 8th/136]
ha

Fourth Heart Sound (S4)


–– Harrison 19th/1448
IC

•• The fourth heart sound (S4) occurs during the atrial filling phase of ventricular diastole and indicates LV presystolic expansion.
•• S4 is more common among patients who derive significant benefit from the atrial contribution to ventricular filling, such as those
with chronic LV hypertrophy or active myocardial ischemia.
PG

•• S4 is not present with atrial fibrillation.


Fourth Heart Sound (S4)
–– Braunward 8th/136
•• It is a low-pitched, presystolic sound produced in the ventricle during ventricular filling; it is associated with an effective atrial
contraction and is best heard with the bell piece of the stethoscope.
•• The sound is absent in patients with atrial fibrillation.
•• The S4 occurs when diminished ventricular compliance increases the resistance to ventricular filling; it is frequently present in
patients with systemic hypertension, aortic stenosis, hypertrophic cardiomyopathy, ischemic heart disease, and acute mitral regurgitation.
•• The right-sided S4 is present in patients with right ventricular hypertrophy secondary to either pulmonic stenosis or pulmonary
hypertension and frequently accompanies a prominent presystolic a wave in the JVP.
•• The incidence of an audible S4 increases with increasing age. Whether an audible S4 in adults without other evidence of cardiac
disease is abnormal remains controversial.

Table (Davidson 22nd/560): Normal and abnormal heart sounds

Sound Timing Characteristics Mechanisms Variable features


First heart Onset of systole Usually single of Closure of mitral and tricuspid Loud: hyperdynamic circulation (anaemia, pregnancy,
sound (S1) narrowly split valves thyrotoxicosis); mitral stenosis Answers
Soft: heart failure; mitral regurgitation &
Explanations
Contd...

247
PGI Chandigarh Self-Assessment & Review: 2017–2013

Sound Timing Characteristics Mechanisms Variable features


Second heart End of systole Split on Closure of aortic and Fixed wide splitting with atrial septal defect
sound (S2) inspiration pulmonary valve Wide but variable splitting with delayed right heart
Single of A2 first emptying (e.g. right bundle branch block)
expiration (p. P2 second Reversed splitting due to delayed left heart emptying
532) (e.g. left bundle branch block)
Third heart Early in diastole, Low pitch, often From ventricular wall due Physiological: yound people, pregnancy
sound (S3) just after S2 heard as ‘gallop’ to abrupt cessation of rapid Pathological: heart failure, mitral regurgitation
filling
Fourth heart End of diastole, Low pitch Ventricular origin (stiff Absent in atrial fibrillation
sound (S4) just before S1 ventricle and augmented A feature of severe left ventricular hypertrophy (e.g.
atrial contraction) related to hypertrophic cardiomyopathy)
atrial filling
Systolic clicks Early or mid- Brief, high- Valvular aortic stenosis Click may be lost when stenotic valve becomes
systole intensity sound Valvular pulmonary stenosis thickened or calcified
Floppy mitral valve Prosthetic clicks lost when valve obstructed by

e
Prosthetic heart sounds thrombus or vegetations
from opening and closing

8/
of normally functioning
mechanical valves
Opening snap Early in diastole High pitch, brief Opening of stenosed leaflets Moves closer to S2 as mitral stenosis becomes more

h,
(OS) duration of mitral valve severs. May be absent in calcific mitral stenosis
Prosthetic heart sounds
ar
143. Ans:  (a) Narrow-complex…, (c) Associated with 2 : 1, 3 : 1 or 4 :…, (d) Best therapy is catheter…, (e) Occur due to macro…
ig
[Ref: Harrison 19th/1484-85; CMDT 2016/393; Davidson 22nd/564]
P wave is absent in atrial fibrillation-The ECG by Hampton 8th/78
nd

Atrial Flutter
–– Davidson 22nd/564
ha

•• Atrial flutter is characterised by a large (macro) re-entry circuit, usually within the right atrium encircling the tricuspid annulus.
•• The atrial rate is approximately 300/min, and is usually associated with 2 : 1, 3 : 1 or 4 : 1 AV block (with corresponding heart rates
of 150, 100 or 75/min).
IC

•• Rarely, in young patients, every beat is conducted, producing a rate of 300/min and, potentially, haemodynamic compromise. The
ECG shows sawtoothed flutter waves. When there is regular 2 : 1 AV block, it may be difficult to identify flutter waves that are buried
in QRS complexes and T waves.
•• Atrial flutter should always be suspected when there is a narrow-complex tachycardia of 150/min. Carotid sinus pressure or
PG

intravenous adenosine may help to establish the diagnosis by temporarily increasing the degree of AV block and revealing flutter
waves
•• Digoxin, β-blockers or verapamil can control the ventricular rate. However, in many cases, it may be preferable to try to restore sinus
rhythm by direct current (DC) cardioversion or by using intravenous amiodarone.
•• Catheter ablation offers a 90% chance of complete cure and is the treatment of choice for patients with persistent symptoms.
“The atrial rate is typically 240–300 beats/min but may be slower in the presence of atrial disease or antiarrhythmic drugs. It often
conducts to the ventricles with 2:1 AV block, creating a regular tachycardia at 150 beats/min, with p waves that may be difficult to
discern. For recurrent episodes of common atrial flutter, catheter ablation of the cavotricuspid isthmus abolishes the arrhythmia in over
90% of patients”- Harrison 19th/1484-85
Essentials of Diagnosis
–– CMDT 2016/393
•• Usually regular heart rhythm.
•• Often tachycardic (100–150 beats/min).
•• Often associated with palpitations (acute onset) or fatigue (chronic).
•• ECG shows “sawtooth” pattern of atrial activity in leads II, III, and AVF.
•• Often seen in conjunction with structural heart disease or chronic obstructive pulmonary disease (COPD).

Answers
&
Explanations

248
November  | 2016

144. Ans:  (b) Downsloping depression of the ST segment Table (Exercise testing): Davidson 22nd/534
>0.1 mV below baseline
Indications
[Ref: Harrison 19th/1582; Davidson 22nd/534]
• To confirm the diagnosis of angina
“If the ST segments are depressed but upward sloping, there is • To evaluate stable angina
probably no ischemia”-ECG by Hampton 8th/144 • To assesses prognosis following myocardial infarction
Stress Testing • To assess outcome after coronary revascularization, e.g. coronary
angioplasty
–– Harrison 19th/1582 • To diagnose and evaluate the treatment of exercise-induced
•• Exercise duration is usually symptom-limited, and the arrhythmias
test is discontinued upon evidence of chest discomfort,
severe shortness of breath, dizziness, severe fatigue, ST- High-risk findings
segment depression >0.2 mV (2 mm), a fall in systolic blood • Low threshold for ischaemia (i.e within stage 1 or 2 of the Bruce
pressure >10 mmHg, or the development of a ventricular protocol)
tachyarrhythmia. • Fall in BP on exercise
•• This test is used to discover any limitation in exercise per- • Widespread, marked or prolonged ischaemic ECG changes
formance, detect typical ECG signs of myocardial ischemia, • Exercise-induced arrhythmia
and establish their relationship to chest discomfort.

e
•• The ischemic ST-segment response generally is defined as 145. Ans:  (d) Aspirin, (e) Recombinant tissue plasminogen…
flat or downsloping depression of the ST segment >0.1 mV

8/
[Ref: Harrison 19th/2561-63; CMDT 2016/981; Davidson 22nd/ 1244]
below baseline (i.e., the PR segment) and lasting longer
than 0.08 s. “Acute Ischemic Stroke:In patients not eligible for thrombolytic
•• Upsloping or junctional ST-segment changes are not therapy, and in whom hemorrhage has been excluded by CT,

h,
considered characteristic of ischemia and do not constitute the immediate administration of aspirin 325 mg orally daily is
a positive test. ar indicated. Anticoagulant drugs are started without delay in the
•• Although T-wave abnormalities, conduction disturbances, setting of atrial fibrillation or other source of cardioembolism
and ventricular arrhythmias that develop during exercise when hemorrhage has been excluded by CT. Treatment is with
ig
should be noted, they are also not diagnostic. warfarin (target INR 2.0–3.0) or dabigatran (150 mg twice
•• Negative exercise tests in which the target heart rate (85% daily); bridging warfarin with heparin is not necessary, but
nd

of maximal predicted heart rate for age and sex) is not some experts advocate treatment with aspirin until the INR
achieved are considered nondiagnostic. becomes therapeutic. Aspirin therapy should not continue after
•• The normal response to graded exercise includes progressive achieving therapeutic anticoagulation because of an increased
ha

increases in heart rate and blood pressure. Failure of risk of hemorrhage”-CMDT 2016/981
the blood pressure to increase or an actual decrease with “Thrombolytic agents are- Streptokinase, urokinase and
signs of ischemia during the test is an important adverse recombinant tissue plasminogen activator(rt-PA e.g alteplase,
IC

prognostic sign, since it may reflect ischemia-induced reteplase, tenecteplase)”-KDT 7th/627


global left ventricular dysfunction. “Streptokinas: It is not employed now except for consideration
•• The development of angina and/or severe (>0.2 mV) of cost. Compared to newer more fibrin-specifc tissue
ST-segment depression at a low workload, i.e., before
PG

plasminogen activators(e.g alteplase) it is less effective in


completion of stage II of the Bruce protocol, and/or
opening occluded coronary arteries and cause less reduction in
ST-segment depression that persists >5 min after the
MI related mortality. There are several other disadvantages as
termination of exercise increases the specificity of the test
and suggests severe IHD and a high risk of future adverse well with streptokinase”- KDT 7th/627
events. Acute Ischemic Stroke
Exercise (Stress) ECG –– Harrison 19th/2561-63
–– Davidson 22nd/534 •• Treatments designed to reverse or lessen the amount of
•• Exercise electrocardiography is used to detect myocardial tissue infarction and improve clinical outcome fall within
ischaemia during physical stress and is helpful in the six categories: (1) medical support, (2) IV thrombolysis,
diagnosis of coronary artery disease. (3) endovascular revascularization, (4) antithrombotic
•• A 12-lead ECG is recorded during exercise on a treadmill or treatment, (5) neuroprotection, and (6) stroke centers and
bicycle ergometer. rehabilitation.
•• The Bruce Protocol is the most commonly used for testing. •• Intravenous thrombolysis: Treatment with IV rtPA within
BP is recorded and symptoms assessed throughout the test. 3 h of the onset of ischemic stroke improved clinical out-
•• A test is ‘positive’ if anginal pain occurs, BP falls or fails come.
to increase, or if there are ST segment shifts of more than •• Platelet Inhibition: Aspirin is the only antiplatelet agent
1 mm . that has been proven effective for the acute treatment of
•• False-negative results can occur in patients with coronary ischemic stroke. The use of aspirin within 48 h of stroke Answers
artery disease, and some patients with a positive test will not onset reduced both stroke recurrence risk and mortality &
Explanations
have coronary disease (false-positive). minimally

249
PGI Chandigarh Self-Assessment & Review: 2017–2013

•• Anticoagulation: Numerous clinical trials have failed to Treatment Target group Approx. NNT to
demonstrate any benefit of anticoagulation in the primary proportion prevent 1
treatment of atherothrombotic cerebral ischemia. Several of patients death or
trials have investigated antiplatelet versus anticoagulant eligible for disability
medications given within 12–24 h of the initial event. The treatment in those
U.S. Trial of Organon 10172 in Acute Stroke Treatment treated
(TOAST), an investigational low-molecular-weight heparin Decompres- Large cerebral < 1% 2
(LMWH), failed to show any benefit over aspirin. Use of sive hemi- infarction
SC unfractionated heparin versus aspirin was tested in IST. craniectomy
Heparin given SC afforded no additional benefit over aspirin Stroke unit Acute stroke 80% 20
and increased bleeding rates. Several trials of LMWHs have care
also shown no consistent benefit in AIS. Furthermore, trials
NNT-number needed to treat
generally have shown an excess risk of brain and systemic
hemorrhage with acute anticoagulation. A recent meta- 146. Ans:  (b) Increase in unconjugated bilirubin, (c) Increase in
analysis of all forms of heparin found no benefit for acute urine... (d) Increase in faecal stercobilinogen
stroke patients at high or low risk of thrombotic events.
Therefore, trials do not support the use of heparin or other [Ref: Harrison 19th/649-50 ; Davidson 22nd/1026-31]

e
anticoagulants for patients with atherothrombotic stroke. Investigation Results Indicating Active Haemolysis

8/
“Thrombolysis: If with recombinant tissue plasminogen
–– Davidson 22nd/1026
activator (rt-PA) is given within 4.5 hours of symptom onset to
carefully selected patients, the haemorrhagic risk is offset by an Hallmarks of Haemolysis

h,
improvement in overall outcome. The earlier treatment is given, •• ↓Haemoglobin
the greater the benefit”- Davidson 22nd/ 1244 ar •• ↑Unconjugated bilirubin
“Aspirin: In the absence of contraindications, aspirin (300 mg •• ↑Lactate dehydrogenase
daily) should be started immediately after an ischaemic stroke •• ↑Reticulocytes
unless rt-PA has been given, in which case it should be withheld •• ↑Urinary urobilinogen
ig
for at least 24 hours. Aspirin reduces the risk of early recurrence Additional Features of Intravascular Haemolysis
and has a small but clinically worthwhile effect on long-term
nd

•• ↓Haptoglobin
outcome”- Davidson 22nd/ 1244 •• ↑Methaemalbumin
“Heparin: Anticoagulation with heparin has been widely used to •• Positive urinary haemosiderin
treat acute ischaemic stroke in the past. Whilst it reduces the risk
ha

•• Haemoglobinuria
of early ischaemic recurrence and venous thromboembolism,
Haemolytic Anaemia
it increases the risk of both intracranial and extracranial
–– Davidson 22nd/1026-31
IC

haemorrhage. Furthermore, routine use of heparin does not


result in better long-term outcomes, and therefore it should not •• Haemolysis indicates that there is shortening of the normal
be used in the routine management of acute stroke. It is unclear red cell lifespan of 120 days.
whether heparin might provide benefit in selected patients, such •• Red cell destruction overloads pathways for haemoglobin
PG

as those with recent myocardial infarction, arterial dissection breakdown in the liver, causing a modest rise in unconju-
or progressing strokes. Intracranial haemorrhage must be gated bilirubin in the blood and mild jaundice.
excluded on brain imaging before considering anticoagulation”- •• Increased reabsorption of urobilinogen from the gut results
Davidson 22nd/ 1244 in an increase in urinary urobilinogen.
•• Red cell destruction releases LDH into the serum.
Table (Davidson 22nd/ 1244): Role of Treatments in Acute Stroke •• The bone marrow compensation results in a reticulocytosis,
and sometimes nucleated red cell precursors appear in the
Treatment Target group Approx. NNT to
blood.
proportion prevent 1
of patients death or Intravascular Hemolysis
eligible for disability •• Whatever the mechanism, intravascular hemolysis is
treatment in those manifested by (1) hemoglobinemia, (2) hemoglobinuria,
treated (3) jaundice, and (4) hemosiderinuria.
Aspirin Acute ischaemic 90% 80 •• Free hemoglobin in plasma is promptly bound by an
stroke α2-globulin (haptoglobin), producing a complex that is
Thrombolysis Acute ischaemic rapidly cleared by the mononuclear phagocyte system,
with rt-PA stroke 10% 9
thus preventing excretion into the urine. Decreased serum
Treatment within 3 10% 20 haptoglobin is characteristic of intravascular hemolysis.
Answers hrs of onset •• When the liver is normal, jaundice is rarely severe. Excessive
& Treatment within bilirubin excreted by the liver into the gastrointestinal tract
Explanations
3–4.5 hrs of onset leads to increased formation and fecal excretion of urobilin

250
November  | 2016

Table (PJM 20th/20): Differential diagnosis of jaundice

Hepatocellular Obstructive Hemolytic


I. History
1. Abdominal pain Absent Present Present in crisis
2. Pruritus Transient Marked Absent
3. Past history (a) Contact with jaundice patient (a) Pain (Stones) (a) Of crisis
(b) Drugs (b) Drugs, blood transfusion, etc.
II. Examination
1. Tender liver May be present Absent Absent
2. Spleen May be present Absent Present
3. Gallbladder Not palpable Palpable if due to neoplasm Not palpable
4. Pallor Absent Present Present
III. Investigations
1. Urine:
Bilirubin Present Present Absent

e
Urobilinogen Present Absent Present

8/
2. Stools: Present Absent Present
Stercobilinogen
3. Peripheral smear Leucopenia in infective hepatitis Normal Reticulocytosis, Spherocytosis

h,
4. LFT
(a) Bilirubin ++ ++ ar +
(b) Alkaline phosphate Raised Markedly raised Normal
(c) SGOT Markedly raised Raised Normal
ig
5. Barium meal and Normal May reveal pancreatic growth Normal
cholangiography
nd

6. RBC Survival Normal Normal Decreased


ha

147. Ans:  (b) Self-limited course in acute form, (d) Immune-mediated destruction of platelets
[Ref: Robbins 9th/Vol.I 658-59; Harrison 19th/72-29]
IC

Immune Thrombocytopenic Purpura (ITP; a/k Idiopathic Thrombocytopenic purpura)


–– Harrison 19th/72-29
•• It is an acquired disorder in which there is immune-mediated destruction of platelets and possibly inhibition of platelet release from
PG

the megakaryocyte.
•• In children, it is usually an acute disease, most commonly following an infection, and with a self-limited course.
•• In adults, it is a more chronic disease, although in some adults, spontaneous remission occurs, usually within months of diagnosis.
•• ITP is termed secondary if it is associated with an underlying disorder; autoimmune disorders, particularly systemic lupus
erythematosus (SLE), and infections, such as HIV and hepatitis C, are common causes. The association of ITP with Helicobacter
pylori infection is unclear.
•• ITP is characterized by mucocutaneous bleeding and a low, often very low, platelet count, with an otherwise normal peripheral blood
cells and smear.
•• Patients usually present either with ecchymoses and petechiae, or with thrombocytopenia incidentally found on a routine CBC.
•• Mucocutaneous bleeding, such as oral mucosa, gastrointestinal, or heavy menstrual bleeding, may be present. Rarely, life-threaten-
ing, including central nervous system, bleeding can occur.
•• Wet purpura (blood blisters in the mouth) and retinal hemorrhages may herald lifethreatening bleeding.
•• Bone marrow examination can be reserved for those who have other signs or laboratory abnormalities not explained by ITP or
in patients who do not respond to initial therapy. The peripheral blood smear may show large platelets, with otherwise normal
morphology. Depending on the bleeding history, iron-deficiency anemia may be present.
•• Laboratory testing is performed to evaluate for secondary causes of ITP and should include testing for HIV infection and hepatitis
C (and other infections if indicated).
Answers
&
Explanations

251
PGI Chandigarh Self-Assessment & Review: 2017–2013

148. Ans:  (a) In DIC both PT and aPTT increase, (d) Hemophilia so called because a tuft of capillary formations is present,
A is inherited as X-linked recessive producing a network, or web, that spans the lumens of dilated
[Ref: CMDT 2016/549,553;Robbins 9th/661-64; Harrison 19th/ 732-38; thin-walled, small arteries and may extend outside the vessel.
Davidson 22nd/1050-56] Plexiform lesions are most prominent in idiopathic and
familial pulmonary hypertension (group 1), unrepaired
“Hemophilia A: Inheritance is X-linked recessive,leading to
affected males and carrier females”- CMDT 2016/553 congenital heart disease with left-to-right shunts (group
“In early Disseminated intravascular coagulation(DIC), the 2), and pulmonary hypertension associated with human
platelet count and fibrinogen levels may remain within the immunodeficiency”- Robbins 9th/700
normal range, albeit reduced from baseline levels. There is Pulmonary Hypertension
progressive thrombocytopenia (rarely severe), prolongation –– Robbins 9th/700
of the activated partial thromboplastin time (aPTT) and •• Based on underlying mechanisms, the World Health
prothrombin time (PT), and low levels of fibrinogen. D-dimer
Organization has classified pulmonary hypertension into
levels typically are elevated due to the activation of coagulation
five groups. These groups are: (1) pulmonary arterial
and diffuse cross-linking of fibrin. Schistocytes on the blood
hypertension, a diverse collection of disorders that all
smear, due to shearing of red cells through the microvasculature,
are present in 10–20% of patients”- CMDT 2016/549 primarily impact small pulmonary muscular arteries;
“Factor XI (FXI) deficiency, also called hemophilia C, plasma (2) pulmonary hypertension secondary to left-heart

e
thromboplastin antecedent deficiency and Rosenthal syndrome. failure; (3) pulmonary hypertension stemming from
FXI deficiency is inherited in an autosomal recessive pattern, lung parenchymal disease or hypoxemia; (4) chronic

8/
meaning both parents must carry the gene to pass it on to thromboembolic pulmonary hypertension;
their children; men and women are affected equally”-www. •• As can be gathered from the classification above, pulmonary
hemophilia.org/Bleeding-Disorders hypertension has diverse causes. It is most frequently

h,
“Mutations in the factor XI gene cause the congenital associated with structural cardiopulmonary conditions
deficiency of factor XI clotting activity. The inheritance pattern ar that increase pulmonary blood flow, pulmonary vascular
of factor XI is autosomal but not completely recessive, because resistance, or left heart resistance to blood flow. Some of the
heterozygotes may have bleeding. more common causes are :Chronic obstructive or interstitial
ig
“DIC: Common findings include the prolongation of PT lung diseases (Group 3),Antecedent congenital or acquired
and/or aPTT; platelet counts μ100,000/μL, or a rapid decline in heart disease Group 2), Recurrent thromboemboli (Group
nd

platelet numbers; the presence of schistocytes (fragmented red 4), Autoimmune diseases (Group 1) and Obstructive sleep
cells) in the blood smear; and elevated levels of FDP. The most apnea(also group 3)
sensitive test for DIC is the FDP level”-Harrison 19th/ 737
ha

Hemophilia B (Christmas Disease, Factor IX Deficiency): 150. Ans:  (b) Child-Pugh score C
As with hemophilia A, the PTT is prolonged and the PT is
normal. Diagnosis of Christmas disease is possible only by [Ref: Harrison 19th/1995; Davidson 22nd/978; www.ncbi.nlm.nih.gov]
IC

assay of the factor levels. The disease is treated with infusions Surgery in the Patient with Liver Disease
of recombinant factor IX”- Robbins 9th/663
–– www.ncbi.nlm.nih.gov
“In severe haemophilia A, bleeding episodes should be
treated by raising the factor VIII level, usually by intravenous •• In a patient with liver disease, surgical risk depends on the
PG

infusion of factor VIII concentrate”- Davidson 22nd/1051-52 severity of liver disease, nature of the surgical procedure
and presence of comorbid conditions. There are a number
Table (Davidson 22nd/1051): Congenital Causes of coagulopathy of liver-related contraindications to elective surgery
•• Once liver disease is identified in a patient in need of
X-linked
surgery, an assessment of the severity of liver disease should
• Haemophilia A and B be undertaken, as should an evaluation for other known
risk factors for perioperative mortality. Data from studies
Autosomal of patients with cirrhosis suggest that the severity of liver
• Von willebrand disease • Combined V and VIII disease can best be assessed by the Child-Turcotte-Pugh
• Factor II, V, VII, X, XI and XIII deficiency (CTP) score (Child, or Child-Pugh, class) and MELD score
deficiencies • Hypofibrinogenaemia •• In general, patients with compensated cirrhosis who have
• Combined II, VII, IX and X deficiency • Dysfibrinogenaemia normal synthetic function have a low overall risk, and the
risk increases for patients with decompensated cirrhosis.
149. Ans:  (c) Familial pulmonary HTN, (d) Congenital heart dis- •• Two of the most important studies, separated by 13 years,
ease with left-to... (e) Pulmonary hypertension… reported nearly identical results: mortality rates for patients
[Ref: Robbins 9th/700 ; Harshmohan 7th/447-48; Harrison 19th/1658 ; undergoing surgery were 10% for those with Child class A,
Davidson 22nd/724] 30% for those with Child class B, and 76–82% for those
with Child class C cirrhosis.
Answers “Morphology of pulmonary hypertension: One extreme in
•• In addition to predicting perioperative mortality, the
& the spectrum of pathologic changes is the plexiform lesion,
Explanations Child class correlates with the frequency of postoperative

252
November  | 2016

complications, which include liver failure, worsening Diagnosis of Multiple Sclerosis


encephalopathy, bleeding, infection, renal failure, hypoxia –– CMDT 2016/1008
and intractable ascites. •• Multiple sclerosis should not be diagnosed unless there is
•• A general consensus is that elective surgery is well tolerated evidence that two or more different regions of the central
in patients with Child class A cirrhosis, permissible with white matter (dissemination in space) have been affected at
preoperative preparation in patients with Child class different times (dissemination in time). The diagnosis may
B cirrhosis (except those undergoing extensive hepatic be made in a patient with two or more typical attacks and
resection or cardiac surgery), and contraindicated in two or more MRI lesions.
patients with Child class C cirrhosis •• To fulfill the criterion of dissemination in space in a patient
“The Child-Pugh score is calculated by adding the scores of the with only one lesion, repeat imaging in a few months
five factors and can range from 5–15. Child-Pugh class is either should demonstrate at least one lesion in at least two of four
A (a score of 5–6), B (7–9), or C (10 or above). Decompensation typical sites (periventricular, juxtacortical, infratentorial,
indicates cirrhosis with a Child-Pugh score of 7 or more (class or spinal); alternatively, an additional attack localized to a
B). This level has been the accepted criterion for listing for liver different site suffices.
transplantation”- Harrison 19th/1995 •• Primary progressive disease requires at least a year of
progressive disease, plus two of three of the following: at
Table ( www.ncbi.nlm.nih.gov): Factors predictive of perioperative
least one typical brain lesion, at least two spinal lesions, or

e
complications and of postoperative mortality
oligoclonal banding in the cerebrospinal fluid

8/
Predictor of complications Predictor of mortality Multiple Sclerosis (MS)
Child-pugh class B and C Male gender –– Harrison 19th/2661-67
•• It is an autoimmune disease of the CNS characterized by

h,
Ascites Child-Pugh class B and C
chronic inflammation, demyelination, gliosis (scarring),
Etiology of cirrhosis other than Etiology of cirrhosis other ar and neuronal loss; the course can be relapsing-remitting or
primary biliary cirrhosis than primary biliary cirrhosis progressive.
Elevated creatinine Ascites •• There is no definitive diagnostic test for MS. Diagnostic
ig
criteria for clinically definite MS require documentation of
Preoperative infection Preoperative infection
two or more episodes of symptoms and two or more signs
nd

Chronic obstructive pulmonary Respiratory surgery that reflect pathology in anatomically noncontiguous
disease white matter tracts of the CNS
Preoperative upper gastrointestimal American Society of •• Symptoms must last for >24 h and occur as distinct episodes
ha

bleed Anaesthesiologists physical that are separated by a month or more.


status IV and V •• MRI or the simultaneous presence of both an enhancing
Invasiveness of surgical procedure lesion and a nonenhancing lesion in an asymptomatic
IC

location. In patients whose course is progressive from


Intraoperative hypotension (20% de-
onset for ≥6 months without superimposed relapses,
crease of base line blood pressure line
blood pressure for 10 min or more) documentation of intrathecal IgG synthesis may be used to
PG

support a diagnosis of PPMS.


American society of Anaesthesiolo-
gists physical status IV and V Partial Table ( Harrison 19th/2665): Diagnostic criteria for multiple
sclerosis (MS)
151. Ans:  
(b) May produce…, (c) Autoimmune inflammatory
condition, (d) Oligoclonal bands may…, (e) Spinal cord Clinical Presentation Additional Data Needed for MS
involvement … Diagnosis

[Ref: Harrison 19th/2661-67; CMDT 2016/1007-09; Davidson 2 or more attacks, objective None
22nd/1188-92; Robbins 9th/1284-85] Clinical evidence of 2 or
more lesions or objective
“Elevated IgG in cerebrospinal fluid and discrete bands of IgG clinical evidence of 1 lesion
(oligoclonal bands) are present in many patients. The presence with reasonable historical
of such bands is not specific, however, since they have been evidence of a prior attack
found in a variety of inflammatory neurologic disorders and 2 or more attacks; objective Dissemination in space,
occasionally in patients with vascular or neoplastic disorders of clinical evidence of 1 lesion demonstrated by
the nervous system”-CMDT 2016/1008 • > 1 T2 lesion on MRI in at least 2
Multiple Sclerosis -lesion size of 1-25 (Majority b/w 5 and out of 4 MS-typical regions of the
10) mm CNS (periventricular, juxtacortical,
infratentorial, or spinal cord) OR
–– Danhert Radiology Manual 7th/314 • Await a further clinical attack Answers
•• Large lesions may masquerade as brain tumours implicating a different CNS site &
Explanations
•• Mass effect/edema in active lesion (infrequent)

253
PGI Chandigarh Self-Assessment & Review: 2017–2013

Table (Harrison 19th/2663): Initial symptoms of MS How Stem Cells can help in Liver Disease Treatment?
–– www.startstemcells.com
Symptom Percentage Symptom Percentage
•• As far back as 2000, researchers showed that hepatocytes
of cases of cases
could grow in the body on non-liver cell sources. This
Sensory loss 37 Lhermitte 3 phenomenon is called transdifferentiation.
Optic neuritis 36 Pain 3 •• Today, autologous (from the patient) adipose tissue stem
Weakness 35 Dementia 2 cells are the only stem cells that have been used clinically
for treating liver disease. The major advantage with using
Paresthesias 24 Visual loss 2 these cells is that because they come from the patient, there
Diplopia 15 Facial palsy 1 is no risk of rejection when they are transplanted back.
Ataxia 11 Impotence 1 •• The stem cells are transdifferentiating into hepatocytes as well
as producing soluble factors that promote regeneration and
Vertigo 6 Myokymia 1
repair. There is also the possibility that the stem cells may be
Paroxysmal attacks 4 Epilepsy 1 fusing with resident hepatocytes to direct their regeneration.
Bladder 4 Falling 1 •• Mesenchymal stem cells are found throughout the adult
body in tissues such as bone, muscle, cartilage and fat.
152. Ans:  (c) Transplanting mesenchymal stem cell from adipose •• Mesenchymal stem cells are among the most ‘multipotent’

e
tissue to liver stem cells that remain in our bodies after birth. This means

8/
that they are still able to make a variety of different cell types.
[Ref: www.hindawi.com/journals/ijh/2012/307165/; Harrison
19th/90e1-3; L and B 26th/1427-29; Schwartz 9th/1124-28] •• Many trials have shown that patients with liver cirrhosis
have benefitted from autologous adipose tissue derived

h,
√ Safer approach meAns:  Lesser chance of rejection, lesser mesenchymal stem cells. We have proven results reversing
transmission of disease and lesser risk of cancerous growth the effects of hepatitis, cirrhosis and liver damage due to
Among all options c, is best
ar chemo and other drug therapies.
Option a More regenerative capacity, so more chance of
Advantages of Stem Cell therapy in Swiss Medica Clinics
abnormal growth (including tumour formation)
ig
Allograft risk for Liver Disease
–– www.startstemcells.com/
nd

Option b Hepatocyte carries allograft risk, risk of transmission


of donor disease •• Side effects and rejection free (use the patient’s very own
stem cells from their abdomen)
Option d Eryhropoietin is used in haematopietic tissue •• Avoidance of any allergic and immune reactions (own cells
ha

generation
suit chromosomal and genetic structure)
Option c Autograft so less immune risk •• No risk of contamination by transmissible diseases
No transmission of disease from donor •• No oncological complication as adult stem cells in the com-
IC

Liver Transplantation parison with embryonic cells are rather mature


–– Harrison 19th/90e-3 •• Small quantity of fat tissue is needed (that is why after
•• Clinical trials of hepatocyte transplantation demonstrate activation process the stem cells can be at once injected to
PG

its potential as a substitute for organ transplantation, but patient without necessity of growing them on substances.
this approach is limited by the paucity of available cells. Embryonic stem cells are grown during several months)
•• Potential sources of stem cells for regenerative strategies •• The period of time between getting lipoaspirate and
include endogenous liver stem cells (such as oval cells), ES injection of the activated stem cells is only a few hours
cells, MSCs, and USCs. •• Adipose tissue stem cells are easily accessible and yielding
•• Although a series of studies in humans as well as animals up to ten times more stem cells than patients bone marrow
suggested that transplanted MSCs and HSCs can General Strategies for Stem cell Replacement
generate hepatocytes, fusion of the transplanted cells with –– Harrison 19th/90e1
endogenous liver cells, giving the erroneous appearance of •• At least three different therapeutic concepts for cell re-
new hepatocytes, appears to be the underlying event in most placement can be envisaged
circumstances. •• One therapeutic approach involves direct administration
•• The available evidence suggests that transplanted HSCs and of stem cells. The cells may be injected directly into the
MSCs can generate hepatocyte-like cells in the liver only at damaged organ or injected systemically
a very low frequency, but there are beneficial consequences •• A second approach involves transplantation of
presumably related to indirect paracrine effects. differentiated cells derived from stem cells.
•• ES cells can be differentiated into hepatocytes and •• A third approach involves stimulation of endogenous stem
transplanted in animal models of liver failure without the cells to facilitate repair. Therapeutic stimulation of precursor
Answers formation of teratomas. Clinical trials are in progress in cells is already a clinical reality in the hematopoietic system,
&
Explanations
cirrhosis with numerous cell types, including MSCs, USCs, where factors such as erythropoietin, granulocyte colony-
HSCs, and ASCs. stimulating factor, and granulocyte-macrophage colony-

254
November  | 2016

stimulating factor are used to increase production of specific induced pluripotent stem (iPS) cells, umbilical-cord blood
blood elements. stem cells (USCs), organ-specific somatic stem cells (e.g.,
•• In addition to these strategies for cell replacement, a neural stem cells for treatment of the brain), and somatic
number of other approaches could involve stem cells for ex stem cells that generate cell types specific for the target
vivo or in situ generation of tissues, a process termed tissue organ rather than the donor organ (e.g., bone marrow
engineering mesenchymal stem cells or CD34+ hematopoietic stem cells
•• Stem cell transplantation is not a new concept but rather is for cardiac repair).
already part of established medical practice. •• Although each cell type has potential advantages and
Embryonic Stem Cells disadvantages, there are a number of generic problems in
developing any of these cell types into a useful and reliable
–– Harrison 19th/90e1
clinical tool
•• Embryonic stem cells have the potential to generate all
the cell types in the body; thus, in theory, there are no
restrictions on the organs that could be regenerated. Hepatocyte Transplantation and Stem Cell Transplantation
•• Cells tend to develop abnormal karyotypes and other –– www.hindawi.com/journals/ijh/2012/307165/
abnormalities with increased time in culture, and ES cells •• The widespread application of hepatocyte transplantation,
have the potential to form teratomas if all cells are not however, is also limited by organ availability, by problems
with viability of isolated hepatocytes after cryopreservation,

e
committed to the desired cell types before transplantation.
Organ-Specific Multipotent Stem Cells and by the potential formation of hepatocyte aggregates

8/
during injection subsequently obstructing liver sinusoids
–– Harrison 19th/90e2
and resulting in portal hypertension or fatal emboli.
•• Cells potentially could be obtained from the patient and
•• The BM compartment is largely made up of HSCs,

h,
amplified in cell culture, circumventing the problems committed progenitor cells, and noncirculating stromal
associated with immune rejection. ar cells called mesenchymal stem cells (MSC) which have the
•• Moreover, these populations of cells are more limited in
ability to develop into mesenchymal lineages .
potentiality than are pluripotent ES or iPS cells, and they •• Unlike hepatocytes, the use of BMSCs for liver regeneration
may be difficult to obtain in large quantities from many
ig
does not depend on the procurement of cadaveric livers
organs. whose donors are often immunologically disparate and
•• A number of early studies of MSCs transplanted into
nd

also in short supply. The use of adult stem cells is attractive


heart, liver, and other organs suggested that the cells had as it overcomes the moral and ethical barriers of ES cell
differentiated into organ-specific cell types with beneficial manipulation.
effects in animal models of disease.
ha

•• Further advantages of the use of BMSCs are that they are


•• Regardless of the source of the stem cells used in regenerative
multipotent, there is already considerable experience in
strategies, a number of generic problems must be overcome their use, they are easily accessible, and there is unlimited
IC

for the development of successful clinical applications. supply. Conversely, concerns have been raised about the
These problems include the devising of methods to adverse long-term effects of stem cell therapy.
reliably generate large numbers of specific cell types, to •• There is evidence to suggest that treatment with BMSCs
minimize the risk of tumor formation or proliferation
PG

may provide liver fibrogenic cells (hepatic stellate cells and


of inappropriate cell types, to ensure the viability and myofibroblasts) which contribute to fibrosis and could have
function of the engrafted cells, to overcome immune a deleterious effect on already decompensated cirrhotic
rejection when autografts are not used, and to facilitate livers. Similarly, there are concerns that hepatocellular
revascularization of regenerated tissue. carcinoma (HCC) originates from hepatic oval cells
Sources of Stem Cells for Tissue Repair and BMSCs. Much of the data concerning the malignant
–– Harrison 19th/90e1 potential of BMSCs, however, originates from genetically
•• A variety of different types of stem cells could be used in modified rodent models and may not be present in humans
regenerative strategies, including embryonic stem (ES) cells,

Table: Autologous Stem cell therapies and clinical studies in Liver disease (Contributing to the hepatocyte population direct cell
administration)

Cell Source Intended Mechanism Evidence Comments


Undifferentiated cell ESCs Hepatic engraftment and Engraftment and Risk of malignancy with ESCs
Adult stem/ hepatocyte differentiation differentiation Clinical trials unrealistic currently
progenitor and function demonstrated but Concerns regarding phenotypic stability
cells limited hepatocyte Answers
function &
Explanations

Contd...

255
PGI Chandigarh Self-Assessment & Review: 2017–2013

Cell Source Intended Mechanism Evidence Comments


Ex vivo differentiated ESCS Hepatic engraftment and Low level of Risk of malignancy with ESCs
hepatocytes Adult stem/ hepatocyte function engraftment with Limited function of ex vivo differentiated
progenitor limited cell function hepatocytes ad present
cells IPSCs Clinical trials
Unrealistic currently
Bioartificial liver support Ex vivo Extracorporeal Improved survival in Further advances required in efficient
(BALS) systems differentiated hepatocytes contact- fulminant/subfulminant hepatocyte generation from human cells
hepatocytes infuse blood via a hepatic failure with Most potential in this field
semiperameable procine-derived
membrane hepatocytes
Promoting endogenous HSCs Matrix remodeling, Improved physiological Therapeutic potential demonstrated
processes vascular remodeling, parameters in phase 1 Further cautious clinical investigation
Bone marrow derived immunomodulation, and clinical trials required
stem cells facilitation of resident
hepatocyte differentiation

e
Bone marrow Matrix remodeling, Improved physical Limited clinical evidence MSC isolation
MSCs immunomodulation, and parameters in a phase problematic Potential for increased hepatic

8/
facilitation and facilitation 1 clilnical trial fibrosis
of resident hepatocyte
differentiation

Surgery
h,
ar •• Secretions and swallowed air are not emptied from the
stomach, and gastric dilation and vomiting may occur.
Return of bowel activity is heralded by the presence of
ig
153. Ans:  (d) Usually resolves within 48–72 hour, (e) Radiographs
bowel sounds, flatus, and bowel movements.
show diffusely dilated bowel with air in the...
In adynamic ileus, the stomach, small bowel, and colon are
nd

••
[Ref: Manipal Surgery 4th/765; L and B 26th/276; Schwartz 9th/992-93; affected.
Sabiston 19th/307-08] •• The diagnosis of bowel obstruction is usually based on
ha

Illeus and Early Postoperative Bowel Obstruction clinical findings and plain radiographs of the abdomen.
•• A CT scan, abdominal radiographs, and small bowel follow-
–– Sabiston 19th/307-08
•• Early postoperative bowel obstruction denotes obstruction
through are variably used to establish the diagnosis and
IC

occurring within 30 days after surgery. The obstruction may assist in treatment decision making.
be functional (i.e., ileus), due to inhibition of propulsive •• In adynamic ileus, abdominal radiographs reveal diffusely
bowel activity, or mechanical as a result of a barrier. dilated bowel throughout the intestinal tract with air in the
PG

•• Ileus that occurs immediately after surgery in the absence colon and rectum.
of precipitating factors and resolves within 2 to 4 days is •• A CT scan is more accurate in differentiating functional
referred to as primary or postoperative ileus. from mechanical obstruction by identifying the so-called
•• On the other hand, ileus that occurs as a result of a transition point or cutoff at the obstruction site in cases of
precipitating factor and is associated with a delay in return mechanical obstruction.
of bowel function is referred to as secondary or adynamic or Paralytic ileus
paralytic ileus. –– L and B 26th/276
•• Mechanical bowel obstruction may be caused by a luminal, •• Paralytic ileus may present with nausea, vomiting, loss of
mural, or extraintestinal barrier. appetite, bowel distension and absence of flatus or bowel
•• In the immediate postoperative period, restricted oral intake movements.
and postoperative narcotic analgesia also contribute to altered •• Following laparotomy, gastrointestinal motility temporarily
small bowel motility. decreases.
•• Mechanical early postoperative small bowel obstruction •• Treatment is usually supportive with maintenance of ad-
is commonly caused by adhesions (92%), a phlegmon
equate hydration and electrolyte levels. However, intestinal
or abscess, internal hernia, intestinal ischemia, or
complications may present as prolonged ileus and so should
intussusception.
be actively sought and treated.
•• Postoperative ileus affects the stomach and colon primarily.
•• Return of function of the intestine occurs in the following
Answers •• After laparotomy, small bowel motility returns within order: small bowel, large bowel and then stomach. This
& several hours, gastric motility within 24 to 48 hours, and
Explanations pattern allows the passage of faeces despite continuing lack
colonic motility in 48 to 72 hours.

256
November  | 2016

of stomach emptying and, therefore, vomiting may continue •• The types of organisms found and the clinical presentations
even when the lower bowel has already started functioning of these two processes are different. In adults, primary
normally. bacterial peritonitis (PBP) occurs most commonly in
conjunction with cirrhosis of the liver (frequently the result
154. Ans:  (a) Marked leukocytosis with left shift…, (b) Usually of alcoholism).
caused by…, (c) May be associated with… •• However, the disease has been reported in adults with
[Ref: Manipal Surgery 4th/634-44; Harrison 19th/846-48; L and B
metastatic malignant disease, postnecrotic cirrhosis, chronic
26th/971-75; Schwartz 9th/124-25; Sabiston 19th/1100-02] active hepatitis, acute viral hepatitis, congestive heart failure,
systemic lupus erythematosus, and lymphedema as well as
Secondary Peritonitis in patients with no underlying disease.
–– Harrison 19th/846-48 •• Although PBP virtually always develops in patients with
•• Secondary peritonitis develops when bacteria contaminate preexisting ascites, it is, in general, an uncommon event,
the peritoneum as a result of spillage from an intraabdominal occurring in ≤10% of cirrhotic patients
viscus(e.g. appendicular perforation).
Table (L and B 25th/973): C linical features in Peritonitis
•• The organisms found almost always constitute a mixed
flora in which facultative gram-negative bacilli and Abdominal pain, worse on movement
anaerobes predominate, especially when the contaminating

e
Guarding/rigidity of abdominal wall
source is colonic.

8/
•• Patients with secondary peritonitis generally have abnormal Pain/tenderness on rectal/vaginal examination (pelvic peritonitis)
findings on abdominal examination, with marked voluntary Pyrexia (may be absent)
and involuntary guarding of the anterior abdominal
Raised pulse rate

h,
musculature. Later findings include tenderness, especially
rebound tenderness. In addition, there may be localized arAbsent or reduced bowel sounds
findings in the area of the inciting event. Septic shock’ [systemic inflammatory response syndrome (SIRS)] in
•• In general, patients are febrile, with marked leukocytosis later stages
and a left shift of the WBCs to band forms.
ig
155. Ans:  (a) Watershed zones…, (b) Arterial ischemic lesion…,
Primary and Secondary Microbial Peritonitis
(c) Crypts may be…, (d) Microscopic examination …
nd

–– Schwartz 9th/124-25
•• Primary microbial peritonitis occurs when microbes invade [Ref: Robbins 9th/779-80; L and B 26th/1173 ;Sabiston 19th/1177]
the normally sterile confines of the peritoneal cavity Ischemic Bowel Disease
ha

via hematogenous dissemination from a distant source


–– Robbins 9th/779-80
of infection or direct inoculation. This process is more
•• Intestinal responses to ischemia occur in two phases.
common among patients who retain large amounts of
IC

The initial hypoxic injury occurs at the onset of vascular


peritoneal fluid due to ascites, and among those individuals
compromise. The second phase, reperfusion injury, is
who are being treated for renal failure via peritoneal
initiated by restoration of the blood supply and it is at this
dialysis. These infections invariably are monomicrobial
time that the greatest damage occurs.
PG

and rarely require surgical intervention.


•• Intestinal segments at the end of their respective arterial
•• Secondary microbial peritonitis occurs subsequent to
supplies are particularly susceptible to ischemia. These
contamination of the peritoneal cavity due to perforation
watershed zones include the splenic flexure, where the
or severe inflammation and infection of an intra-abdominal
superior and inferior mesenteric arterial circulations
organ. Examples include appendicitis, perforation of any
terminate,and, to a lesser extent, the sigmoid colon and
portion of the gastrointestinal tract, or diverticulitis.
rectum where inferior mesenteric, pudendal, and iliac
Peritonitis arterial circulations end.
–– L and B 26th/971-75 •• Although the colon is the most common site of gastrointes-
•• Peritoneal infection is usually caused by two or more tinal ischemia, mucosal and mural infarction may involve
bacterial strains. any level of the gut from stomach to anus.
•• There are also less common forms in which the aetiology •• Substantial portions of the bowel are generally involved in
is a primary ‘spontaneous’ peritonitis, in which a pure transmural infarction due to acute arterial obstruction.
infection with streptococcal, pneumococcal or haemophilus The demarcation between normal and ischemic bowel is
bacteria occurs. sharply defined and the infarcted bowel is initially intensely
Primary (Spontaneous) Bacterial Peritonitis congested and dusky to purple-red. There is coagulative
–– Harrison 19th/846 necrosis of the muscularis propria within 1 to 4 days, and
•• Peritonitis is either primary (without an apparent source of perforation may occur.
Answers
contamination) or secondary. &
Explanations

257
PGI Chandigarh Self-Assessment & Review: 2017–2013

•• In mesenteric venous thrombosis, arterial blood continues •• Alterations in bowel habits are not characteristic of carcinoma
to flow for a time, resulting in a less abrupt transition of the right colon, and obstruction is uncommon.
from affected to normal bowel. However, propagation Carcinoma Left Colon
of the thrombus may lead to secondary involvement of
–– CSDT11th/ 719
the splanchnic bed. The ultimate result is similar to that
•• The left colon has a smaller lumen than the right, and the
produced by acute arterial obstruction because impaired feces are semisolid.
venous drainage eventually prevents oxygenated arterial •• Tumors of the left colon can gradually occlude the lumen,
blood from entering the capillaries. causing changes in bowel habits with alternating constipa-
•• Microscopic examination of ischemic intestine tion and increased frequency of defecation (not true watery
demonstrates the characteristic atrophy or sloughing diarrhea).
of surface epithelium. In contrast, crypts may be •• Partial or complete obstruction may be the initial picture.
hyperproliferative. Inflammatory infiltrates are initially
Colon and Rectal Cancers: HNPCC Association
absent in acute ischemia, but neutrophils are recruited
–– Schwartz 9th/1046
within hours of reperfusion.
•• Hereditary Nonpolyposis Colon Cancer (Lynch’s
Mesenteric Ischemia Syndrome): It is characterized by the development of
–– Sabiston 19th/1177 colorectal carcinoma at an early age (average age, 40–45

e
•• Mesenteric ischemia can be secondary to either acute or years).

8/
chronic arterial or venous insufficiency •• Approximately 70% of affected individuals will develop
•• Acute colonic ischemia is the most common form of colorectal cancer. The risk of synchronous or metachronous
mesenteric ischemia. colorectal carcinoma is 40%.

h,
•• It tends to occur in the watershed areas of the splenic
157. Ans:  (e) Medullary carcinoma is associated with MEN-2b
flexure and the rectosigmoid colon, but can be right-sided ar
in up to 40% of patients. [Ref: Harrison 19th/2305-08;Manipal Surgery 4th/339-45; L and B
•• The diagnosis is generally confirmed with flexible endos- 26th/763-70;Sabiston 19th/904-11]
ig
copy, which reveals edema, hemorrhage, and a demarcation “Histologically, papillary carcinomas may exhibit papillary
between the normal and abnormal mucosa. projections, a mixed pattern of papillary and follicular
nd

•• In 85% of cases, the ischemia is self-limited and resolves structures, or a pure follicular pattern (follicular variant).
without incident. Other variants of papillary carcinoma include tall cell, insular,
columnar, diffuse sclerosing, clear cell, trabecular, and poorly
ha

156. Ans:  (b) May be associated with HNPCC, (c) Left-sided differentiated types. These variants account for about 1% of all
cancer presents…, (d) Left side colon is more commonly papillary carcinomas and are generally associated with a worse
involved prognosis”-Schwartz 9th/1362
IC

“Irradiation to the neck during childhood increases the risk of


[Ref: L and B 26th/ 1163-65; Schwartz 9th/1041-50; CSDT11th/ 719 ]
papillary carcinoma”- Manipal Surgery 4th/339
“Tumours of the left side of the colon which are far more “Follicular carcinoma usually arise in a multinodular goiter
PG

common. It usually present with a change in bowel habit or especially in case of endemic goiter(Iodine deficient areas)”-
rectal bleeding, while more proximal lesions typically present Manipal Surgery 4th/343
later with iron deficiency anaemia or a mass”- L and B 26th/1164
“Follicular carcinomas account for 10% of thyroid cancers and
Table (L and B 25th/ 1180): Symptoms and signs of colorectal cancer occur more commonly in iodine-deficient areas”- Schwartz
9th/1363
• Right-sided tumours: Iron deficiency anaemia, abdominal mass
• Left-sided tumours: Rectal bleeding, alteration in bowel habit, Hürthle Cell Carcinoma
tenesmus, obstruction
–– Sabiston 19th/ 912
• Metastatic disease: Jaundice, ascites, hepatomegaly; other
•• Hürthle cell carcinoma is a subtype of follicular carcinoma
symptoms and signs from rarer sites of metastasis
that closely resembles FTC both grossly and on microscopic
• There may be considerable overlap between these symptoms examination.
Carcinoma Right Colon •• Some studies have suggested that Hürthle cell carcinoma
–– CSDT11th/ 719 may have a worse clinical prognosis than standard FTC;
•• The right colon has a large caliber and a thin and distensible however, there is no uniform agreement on these findings.
wall, and the fecal content is fluid. •• Unlike papillary and follicular cancer, spread to local
•• Patients often see a physician for complaints of fatigue and lymph nodes is a poor prognostic event associated with
weakness due to severe anemia. Unexplained microcytic nearly 70% mortality
Answers hypochromic anemia should always raise the question of •• Higher chance of spread to lymph node compared to
&
carcinoma of the colon. follicular carcinoma-Manipal 4th/347
Explanations

258
November  | 2016

Follicular Carcinoma •• Most patients with Schatzki’s rings present with dysphagia.
–– L and B 26th/765 The dysphagia is usually to solid foods only and comes on
•• These appear to be macroscopically encapsulated but, abruptly with nearly complete obstruction.
microscopically, there is invasion of the capsule and of the •• Diagnosis of a Schatzki’s ring is made with a barium
vascular spaces in the capsular region esophagram.
•• Blood-borne metastases are more common and the eventual Most commonly it presents as intermittent dysphagia, especial-
mortality rate is twice that of papillary cancer. ly to solid food. A history of food impaction is also very com-
•• Hurthle cell tumours are a variant of follicular neoplasm
mon. Dysphagia is more common in patients with an associated
motility disorder”-radiopaedia.org
in which oxyphil (Hurthle, Askanazy) cells predominate
histologically. Hurthle cell cancers are associated with “Pathology: The pathogenesis of the Schatzki ring is unclear
a poorer prognosis and some hold that all Hurthle cell with conflicting hypotheses that include redundant pleats
of mucosa, congenital abnormalities and modified peptic
neoplasms are malignant
strictures. Interestingly, there is a reduced incidence of
Medullary Carcinoma Barrett oesophagus in patients with a Schatzki ring. Schatzki
–– Sabiston 19th/1005-06 rings are located at the gastro-oesophageal junction, illustrated
•• MCT accounts for 3% to 9% of thyroid cancers and arises by the fact that there is squamous epithelium above and
from parafollicular cell, or C cell, derived from the neural columnar epithelial below the ring”- radiopaedia.org

e
crest. Schatzki’s Ring
•• MCT is associated with the secretion of a biologic marker,

8/
–– L and B 26th/1019
calcitonin.
•• Schatzki’s ring is a circular ring in the distal oesophagus,
•• Medullary carcinoma can occur in a sporadic form or as part
usually at the squamocolumnar junction. The cause is

h,
of MEN 2A or 2B. MEN 2A usually has a more favorable
obscure, but there is a strong association with reflux
long-term outcome than MEN 2B or sporadic MCT does ar disease.
Table ( L and B): Major differences between papillary and follicular •• The core of the ring consists of variable amounts of fibrous
carcinoma tissue and cellular infiltrate.
ig
Papillary(%) Follicular (%) Schatzki’s Ring
–– Schwartz 9th/845-46
nd

Male incidence 22 35
•• Schatzki’s ring is a thin submucosal circumferential ring in
Lymph node metastases 35 13
the lower esophagus at the squamocolumnar junction, often
Blood vessel invasion 40 60
ha

associated with a hiatal hernia.


Recurrence rate 19 29 •• Barium esophagogram showing Schatzki’s ring (i.e., a thin
Overall mortality rate 11 24 circumferential ring in the distal esophagus at the squamo-
IC

Location of recurrent carcinoma columnar junction).


Distant metastases 45 75 •• Symptoms associated with Schatzki’s ring are brief episodes
of dysphagia during hurried ingestion of solid foods.
Nodal metastases 34 12
PG

•• Its treatment has varied from dilation alone to dilation with


Local recurrence 20 12 antireflux measures, antireflux procedure alone.
158. Ans:  (b) Concentric symmetric…, (c) Consists of esophageal
mucosa…, (d) Dysphagia is usually…, (e) Association with…
[Ref: Manipal Surgery 4th/451; L and B 26th/1019-20 ; Schwartz 9th/845-
46;Sabiston 19th/1037]

Schatzki Ring
–– Sabiston 19th/1037
•• Lying precisely at the squamocolumnar mucosal GEJ,
this ring consists of a concentric symmetric narrowing
representing an area of restricted distensibility of the lower
esophagus.
•• It consists of esophageal mucosa above and gastric mucosa
below, with variable amounts of muscularis mucosae,
connective tissue, and submucosal fibrosis in between .
•• It does not have a component of true esophageal muscle,
nor is it associated with esophagitis. Fig.:  (L and B 26th/ 1020): Schatzki’s ring, a thin submucosal web Answers
•• It is often accompanied by a small hiatal hernia, and some completely encircling the whole of the lumen, usually situated at &
Explanations
have advocated that it is a result of reflux esophagitis the squamocolumnar junction

259
PGI Chandigarh Self-Assessment & Review: 2017–2013

159. Ans:  (a) Incidence decreased…, (b) Associated with H…, Intestinal Diffuse
(c) More common in male…, (d) Better prognosis…
Increasing incidence with age Younger age group
[Ref: Manipal Surgery 4th/487-90; L and B 26th/1045-48 ; Schwartz
9th/926-33; Sabiston 19th/1204-09] Gland formation Poorly differentiated, signet ring
cells
The Diffuse form of Gastric Adenocarcinoma-linitis Plastica
Hematogenous spread Transmural/lymphatic spread
–– Manipal Surgery 4th/491
•• Poorly differentiated Microsatellite instability Decreased E-cadherin
•• Metastasis early APC gene mutations
•• More common in women p53, p16 inactivation p53, p16 inactivation
•• Intraperitoneal metastases are frequent
•• The prognosis is less favourable for patients with diffuse- Table ( Schwartz 9th/928): Factors increasing or decreasing the risk
subtype histology of gastric cancer
Gastric Carcinoma
Increase risk
–– Schwartz 9th/926-28
Family history
•• Over the past several decades, there has been a dramatic
Diet (high in nitrates, salt, fat)

e
decrease in the gastric cancer incidence and death rate in
most Western industrialized countries. This decrease has Familial polyposis

8/
been largely in the so-called intestinal form rather than in Gastric adenomas
the diffuse form of gastric cancer
Hereditary nonpolyposis colorectal cancer
•• In younger patients, tumors are more often of the diffuse

h,
variety and tend to be large, aggressive, and more poorly Helicobacter pylori infection: Atrophic gastritis, intestinal metaplasia,
dysplasia
differentiated, sometimes infiltrating the entire stomach
(linitis plastic).
arPrevious gastrectomy or gastrojejunostomy (>10 yr ago)
Pathology of Gastric Cancer Tobacco use
ig
–– Sabiston 19th/1206-07 Ménétrier’s disease
•• The Lauren system separates gastric adenocarcinoma into
nd

Decrease risk
intestinal or diffuse types based on histology. Aspirin
•• The intestinal variant typically arises in the setting of a
Diet (high fresh fruit and vegetable intake)
ha

recognizable precancerous condition such as gastric atrophy


or intestinal metaplasia. Men are more commonly affected Vitamin C
than women, and the incidence of the intestinal-type gastric
160. Ans:  ( a) Appendix is the most common leading point,
IC

adenocarcinoma increases with age. The intestinal variety


is well differentiated with a tendency to form glands. (c) Usually presents with pain without vomiting
Metastatic spread is generally hematogenous to distant [Ref: Manipal Surgery 4th/749-53; L and B 26th/114-15]
organs.
PG

•• The intestinal type is also the dominant histology in areas Intussusception in Children
in which gastric cancer is epidemic, suggesting an environ- –– L and B 26th/114-15
mental etiology. •• Most intussusceptions in children are seen from two months
•• An intermediate step in Correa’s model of gastric to two years of age. They are life-threatening.
cancer(intestinal type) development is intestinal metaplasia.
•• Intestinal metaplasia occurs due to persistent irritation •• Intussusception typically causes a strangulating bowel ob-
of the gastric mucosa, most commonly from H. pylori struction, which can progress to gangrene and perforation.
infection. •• Intussusception is classified according to the site of the
•• The risk for progression from intestinal metaplasia to intussusceptum and intussuscipiens. In children, more than
gastric cancer is higher in the type III metaplasia than in 80 per cent are ileocolic, beginning several centimetres
type I proximal to the ileocaecal valve with their apex found in the
ascending or transverse colon.
Table (Sabiston 19th/1207): Lauren classification system
•• In the majority, the cause is hyperplasia of Peyer’s patches
Intestinal Diffuse (lymphoid tissue), which may be secondary to a viral
Environmental Familial infection. In 10 per cent of children, intussusception is
secondary to a pathological lead point, such as a Meckel’s
Gastric atrophy, intestinal Blood type A diverticulum, enteric duplication cyst or even a small bowel
Answers
metaplasia lymphoma. Such cases are more likely in children over the
&
Explanations Men >women Women >men age of two years and in those with recurrent intussusception.

260
November  | 2016

•• Classically, a previously healthy infant presents with colicky •• Most commonly used noninvasive test for diagnosis is
pain and vomiting (milk, then bile). Between episodes, ultrasound-pseudokidney sign, target sign, duplex-assesses
the child initially appears well. Later, they may pass a vascularity also
‘redcurrant jelly’ stool.
•• Clinical signs include dehydration, abdominal distension 161. Ans:  (a) Silk, (c) Polypropylene, (d) Polyester, (e) Nylon
and a palpable sausage-shaped mass in the right upper [Ref: L and B 26th/37-38 ; Sabiston 19th/233]
quadrant.
•• A plain radiograph is rarely requested but if done it Table ( L and B 26th/ 38): Absorbable(Suture--Tensile strength
commonly shows signs of small bowel obstruction and a retention in vivo)
soft-tissue opacity. Diagnosis is confirmed on an abdominal
ultrasound. Catgut (Plain): Lost within 7–10 days
•• After resuscitation with intravenous fluids, broad-spectrum Catgut (Chromic): Lost within 21–28 days
antibiotics and nasogastric drainage, non-operative
Polyglactin: Approximately 60% remains at 2 weeks
reduction is attempted using an air enema .
•• More than 70 per cent of intussusceptions can be reduced Polyglyconate: Approximately 70% remains at 2 weeks
nonoperatively. Polyglycolic: Approximately 40% remains at 1 week 2 weeks
•• If an operative reduction is needed, this is usually performed

e
open. The intussusception is milked distally by gentle Polydioxanone: Approximately 70% remains at 2 weeks

8/
compression from its apex. Both the intussusceptum and Polyglycaprone: 21 days maximum
the intussuscipiens are inspected for areas of non-viability.
•• An irreducible intussusception or one complicated by Table (L and B 26th/ 37): Non-absorbable (Suture- Tensile strength)

h,
infarction or a pathological lead point requires resection and
primary anastomosis. ar Nylon (Polyamide polymer): Loses 15–20% per year
Intussusception Polyester :Infinite (>1 year)
–– Manipal Surgery 4th/749-53 Polybutester :Infinite (>1 year)
ig
•• Most common cause is idiopathic
Polypropylene: Infinite (>1 year)
•• Most common type of non-operative reduction is by using
nd

air and barium enema Table (Sabiston 19th/233): Comparison of absorbable sutures

Suture Types Raw material Tensile strength retention in vivo Tissue reaction
ha

Surgical Gut Suture Chromic Collagen derived from Individual patient characteristics can affect Moderate reaction
healthy beef and sheep rate of tensile strength loss.
Monocryl Suture Monofilament Copolymer of glycolide and ∼50%-60% (violet: 60%-70%) remains at 1 Minimal acute
IC

(poliglecaprone 25) epsilon-caprolactone week. inflammatory


reaction
Coated Vicryl Suture Braided Copolymer of lactide and ∼75% remains at 2 weeks. Minimal acute
PG

(polyglactin 910) Monofilament glycolide coated with 370 ∼50% remains at 3 weeks. inflammatory
and calcium stearate reaction
PDS II Suture Monofilament Polyester polymer ∼70% remains at 2 weeks Slight reaction
(polydioxanone) ∼50% remains at 4 weeks
∼25% remains at 6 weeks

Table (Sabiston 19th/233): Comparison of Nonabsorbable Sutures

Suture Types Raw Material Tensile Strength Retention In Vivo Tissue Reaction
Silk Suture Braided Organic protein called fibroinProgressive degradation of fiber may result Acute inflammatory
in gradual loss of tensile strength over reaction
time.
Nylon Suture Monofilament Long-chain aliphatic polymers Progressive hydrolysis may result in Minimal acute
Nylon 6 or Nylon 6,6 gradual loss of tensile strength over time. inflammatory reaction
Polyester Fiber Braided Poly (ethylene terephthalate) No significant change known to occur in Minimal acute
Suture Monofilament vivo. inflammatory reaction
Polypropylene Monofilament Isotactic crystalline Not subject to degradation or weakening Minimal acute
Suture stereoisomer of by action of tissue enzymes. inflammatory reaction
polypropylene Answers
&
Explanations

261
PGI Chandigarh Self-Assessment & Review: 2017–2013

162. Ans:  (a) Can occurs due to environmental flora in OT, (b) Superficial infection require re-surgical exploration of wound
[Ref: Manipal Surgery 4th/27-28; CSDT 14th/84-90; L and B 26th/50-66; Schwartz 9th/335-36,123-24 ; Sabiston 19th/244-46]
SSI: within 1 year if an implant is left in place (so option e is not correct as it mentioned after 1year)

Definition of SSI
–– CSDT 14th/84-90
•• The Centers for Disease Control and Prevention (CDC) define SSI as an infection that occurs at or near the surgical incision within
30 postoperative days of the surgical procedure, or within 1 year if an implant is left in place (e.g., mesh, heart valve)
•• The CDC further classifies SSI as: 1. Superficial incisional 2. Deep incisional 3. Organ/space SSI

Surgical Site Infection(SSI)


–– Manipal Surgery 4th/27-28
•• Staph. aureus remains the most common pathogen in SSIs
•• Main source of SSIs: Environment(this is from operating room and patient related)
•• Preoperative hair removal (clipping) should be done immediately before an operation

e
Surgical Site Infection(SSI)

8/
–– Sabiston 19th/244-46
•• By definition, they can occur anytime from 0 to 30 days after the operation or up to 1 year after a procedure that has involved the
implantation of a foreign material (mesh, vascular graft, prosthetic joint, and so on).

h,
•• Study has shown that Staphylococcus aureus remains the most common pathogen in SSIs, followed by coagulase-negative staphylococci,
enterococci, and Escherichia coli. ar
•• Microorganisms causing SSI can be either exogenous or endogenous. Exogenous microorganisms come from the operating team or
from the environment around the surgical site (OR, equipment, air, water, and so on).
ig
•• Preoperative shaving has been shown to increase the incidence of SSI after clean procedures as well.
•• In general, the mainstay of treatment is source control or draining of the infected area. For a superficial SSI this involves opening
nd

the wound at the skin and subcutaneous levels and cleansing the wound, along with dressing changes twice or three times a day.
•• For organ/procedure-related SSI, source control can generally be achieved with percutaneous drainage.
“Preoperative hair removal has been correlated in several studies with a higher rate of SSI,even when the procedure involves the scalp
ha

or the patient has abundant hair at the surgical site. A 2006 Cochrane database systematic review and meta-analysis concluded that
preoperative shaving increases the rate of SSI by at least twofold; it is noteworthy that there was no difference in the rates of SSI when
hair clipping was compared to no hair removal. Therefore, if hair removal is deemed necessary, preoperative hair clipping is preferred
IC

to shaving”-CSDT 14th/87

Table (CSDT 14th/85): Criteria and definitions of surgical site infection.


PG

Superficial incisional SSI Deep incisional SSI Organ/space SSI


Infection occurs within 30 d after the Infection occurs within 30 d after the Infection occurs within 30 d after the
operative procedure and Involves only skin operative procedure if no implant is left in operative procedure if no implant is left in
and subcutaneous tissue of the incision and place or within 1 y if implant is in place and place or within 1 y if implant is in place and
Patient has at least one of the following: the infection appears to be related to the the infection appears to be related to the
• Purulent drainage from the superficial operative procedure and operative procedure and
incision. Involves deep soft tissues (eg, fascial and Infection involves any part of the body,
• Organisms isolated from an aseptically muscle layers) of the incision and excluding the skin incision, fascia, or muscle
obtained culture of fluid or tissue from the Patient has at least one of the following: layers, that is opened or manipulated during
superficial incision. • Purulent drainage from the deep incision the operative procedure and
• At least one of the following signs or but not from the organ/space component Patient has at least one of the following:
symptoms of infection: pain or tenderness, of the surgical site. • Purulent drainage from a drain that is
localized swelling, redness, or heat, and • A deep incision spontaneously dehisces or placed through a stab wound into the
superficial incision are deliberately opened is deliberately opened by a surgeon and organ/space.
by surgeon, and are culture-positive or not is culture-positive or not cultured and the • Organisms isolated from an aseptically

Answers
cultured. A culture-negative finding does patient has at least one of the following obtained culture of fluid or tissue in the
& not meet this criterion. signs or symptoms: fever (> 38°C), organ/space.
Explanations

262
November  | 2016

Superficial incisional SSI Deep incisional SSI Organ/space SSI


• Diagnosis of superficial incisional SSI by the or localized pain or tenderness. A culture- • An abscess or other evidence of infection
surgeon or attending physician. negative finding does not meet this involving the organ/space that is found on
criterion. direct examination, during reoperation,
• An abscess or other evidence of infection or by histopathologic or radiologic
involving the deep incision is found on examination.
direct examination, during reoperation, • Diagnosis of an organ/space SSI by a
or by histopathologic or radiologic surgeon or attending physician.
examination.
• Diagnosis of a deep incisional SSI by a
surgeon or attending physician.

•• It is unnecessary and may be detrimental to start them


163. Ans:  (b) Should be given half an hour prior to surgery
more than 1 hour preoperatively, and it is unnecessary to
[Ref: CSDT 14th/88; L and B 26th/59-61; Manipal Surgery 4th/27-28; give them after the patient leaves the operating room.
Sabiston 19th/250-55] •• Prophylactic antibiotic coverage for more than 12 hours for
a planned operation is never indicated.
Table ( CSDT 14th/88): Surgical Care Improvement Project (SCIP)

e
measures aimed at prevention of surgical site infections. Prophylactic Antibiotics

8/
–– L and B 26th/59-61
SCIP Infection Measure 1: Prophylactic antibiotic received within 1 h
•• If antibiotics are given to prevent infection after surgery or
prior to surgical incision
instrumentation, they should be used when local wound

h,
SCIP Infection Measure 2: Appropriate prophylactic antibiotic defences are not established (the decisive period). Ideally,
selection for the surgical patient ar maximal blood and tissue levels should be present at
SCIP Infection Measure 3: Prophylactic antibiotic discontinued within the time at which the first incision is made and before
24 h after surgery end time (within 48 h after cardiac surgery end contamination occurs. Intravenous administration at
ig
time) induction of anaesthesia is optimal.
SCIP Infection Measure 4: Blood glucose level at 6 am < 200 mg/dL on •• In long operations or when there is excessive blood loss or
nd

postoperative days 1 and 2 in cardiac surgery when unexpected contamination occurs, antibiotics may be
SCIP Infection Measure 6: No or appropriate hair removal (clippers repeated at 4-hourly intervals during the surgery, as tissue
not shaving) antibiotic levels often fall faster than serum levels.
ha

SCIP Infection Measure 7: Immediate postoperative normothermia •• The use of the newer, broad-spectrum antibiotics for
(temperature > 96.8°F within 15 min postoperatively) in colorectal prophylaxis should be avoided
surgery
IC

Choice of Antibiotics for Prophylaxis


Prophylactic Antibiotic Therapy –– L and B 26th/61
–– Sabiston 19th/250-55; Sabiston 18th •• Empirical cover against expected pathogens with local
PG

•• “Antibiotic prophylaxis is indicated for most clean-contam- hospital guidelines


inated and contaminated(or potentially contaminated) op- •• Single-shot intravenous administration at induction of
erations. anaesthesia
•• Antibiotic prophylaxis for clean surgery is controversial. •• Repeat only during long operations or if there is excessive
When bone is incised (e.g. craniotomy, sternotomy) or a blood loss
prosthesis is inserted, antibiotic prophylaxis is generally
indicated. Some controversy persists with clean surgery of 164. Ans:  (b) Wound contaminated with gross fecal material
soft tissue(e.g breast, hernia)”- Sabiston 19th/251 spillage
•• Prophylactic antibiotic therapy is clearly more effective [Ref: Schwartz 9th/123; L and B 26th/63; CSDT 14th/86]
when begun preoperatively and continued through the
Appendicular perforation- Type IV
intraoperative period
Incision through abscess- Type IV
•• Antibiotics started as late as 1 to 2 hours after bacterial Surgery over a clean site-Type I
contamination are markedly less effective, and it is Wound in urinary tract without unusual contamination-type II
completely without value to start prophylactic antibiotics
Wound Class
after the wound is closed.
•• The optimal time to give parenteral antibiotic prophylaxis –– Schwartz 9th/123
is within 1 hour prior to incision. Antibiotic given sooner •• Clean wounds (class I) include those in which no infection
are ineffective, as are agents given after incision is closed”- is present; only skin microflora potentially contaminate
Sabiston 9th/252 the wound, and no hollow viscus that contains microbes Answers
&
is entered. Class I D wounds are similar except that a Explanations
prosthetic device (e.g., mesh or valve) is inserted.

263
PGI Chandigarh Self-Assessment & Review: 2017–2013

•• Clean/contaminated wounds (class II) include those in which a hollow viscus such as the respiratory, alimentary, or genitourinary
tracts with indigenous bacterial flora is opened under controlled circumstances without significant spillage of contents.
•• Contaminated wounds (class III) include open accidental wounds encountered early after injury, those with extensive introduction of
bacteria into a normally sterile area of the body due to major breaks in sterile technique (e.g., open cardiac massage), gross spillage
of viscus contents such as from the intestine, or incision through inflamed, albeit nonpurulent tissue.
•• Dirty wounds (class IV) include traumatic wounds in which a significant delay in treatment has occurred and in which necrotic tissue
is present, those created in the presence of overt infection as evidenced by the presence of purulent material, and those created to
access a perforated viscus accompanied by a high degree of contamination.
Table (Schwartz 9th/123): Wound class examples of cases expected infection rates

Wound class Example of class Expected infection rate


Clean (class I) Hernia repair, breast biopsy specimen 1-2%
Clean/contaminated (class II) Cholecystectomy, elective GI surgery (not colon) 2.1%-9.5%
Clean/contaminated (class II) Colorectal surgery 4-14%
Contaminated (class III) Penetrating abdominal trauma, large tissue injury, enterotomy 3.4-13.2%

e
during bowel obstruction

8/
Dirty (class IV) Perforated diverticulitis, necrotizing soft tissue infections 3.1-12.8%

Table (L and B 26th/63): Surgical site infection (SSI) rates relating to wound contamination

h,
Type of surgery Infection rate (%) Rate before prophylaxis

Clean (no viscus opened) 1–2


ar The same
Clean-contaminated (viscus opened, minimal <10 Gastric surgery up to 30% Biliary surgery up to 200%
spillage)
ig
Contaminated (open viscus with spillage or 15–20 Variable but up to 60%
nd

inflammatory disease)
Dirty (pus or perforation, or incision through an ab- <40 Up to 60% or more
scess)
ha

Table (CSDT 14th/86): Wound classification

Clean These are uninfected operative wounds in which no inflammation is encountered and the respiratory, alimentary, genital,
IC

or uninfected urinary tracts are not entered.


Clean/Contaminated These are operative wounds in which the respiratory, alimentary, genital, or urinary tract is entered under controlled
conditions and without unsusual contamination.
PG

Contaminated These include open, fresh, accidental wounds, operations with major breaks in sterile technique or gross spillage from the
gastrointestinal tract, and incisions in which acute, nonpurulent inflammation is encountered.
Dirty These include old traumatic wounds with retained deviatlized tissue and those that involve existing clinical infection or
perforated viscera.

165. Ans:  (a) Seminoma is more…, (b) Cryptorchidism is associated…, (c) Retroperitoneal LN dissection…, (d) Usually presents as…
[Ref: Manipal Surgery 4th/994-99; L and B 26th/1385-87 ; Robbins 9th/975-79;Schwartz 9th/1462 ; Sabiston 19th/1858]
“Seminomas are the most common type of germ cell tumor, making up about 50% of these tumors. The peak incidence is the third
decade and they almost never occur in infants”- Robbins 9th/976
Testicular Tumors
–– CSDT 14th/1008-09
•• Metastases first develop in the retroperitoneal nodes; right-sided tumors metastasize primarily to the interaortocaval region just
below the renal vessels and left-sided tumors primarily to the left para-aortic area at the same level.
•• Testicular tumors present as a painless firm mass within the testicular substance. They often have been present for several months
before the patient seeks consultation. Occasionally (10%), a hydrocele is present
•• Following orchiectomy, three management options are available: (i) active surveillance, (ii) retroperitoneal lymph node dissection
Answers (RPLND), and (iii) systemic chemotherapy
&
Explanations •• RPLND is recommended for clinical stage I patients (no evidence of metastatic disease on imaging) or for those with retroperitoneal
lymphadenopathy that is not bulky (stage IIA-IIB).

264
November  | 2016

Table (CSDT 14th/1009): Treatment and prognosis of testicular cancer related to tumor stage

Conventional stage TNM stage Clinical findings Treatment Five-Year Survival (%)
I T1 Confined to testicle Nonseminoma: RPLND vs. Surveillance; Seminoma: >95
irradiation
IIA N1 Regional nodes Adjuvant chemotherapy >90
<2cm Nonseminoma: RPLND or chemotherapy; Seminoma:
XRT or chemotherapy
IIB N2 Nodes 2-5 cm Adjuvant chemotherapy >85
Nonseminoma; RPLND or adjuvant chemotherapy;
Seminoma: XRT or chemotherapy
IIC N3 Nodes > 5 cm Chemotherapy followed by resection of residual ~70
III M+ Distant metastases disease
Testicular Cancer
–– Schwartz 9th/1462
•• Most men are diagnosed with an asymptomatic enlarging mass.

e
•• A major risk for the development of testicular cancer is cryptorchidism

8/
Testicular Tumour
–– Manipal Surgery 4th/994-99
•• Seminoma is the most common type of germ cell tumour

h,
•• Undescended testis, undoubtedly predisposes to seminoma
“Nonseminomatous germ cell tumours are usually associated with dedescended testes”-Sabiston 19th/1858
ar
Tumours of the Teste
ig
–– L and B 26th/1385-87
•• The peak incidence of seminomas is in the fourth decade of life with the non-seminomatous germ cell tumours being more common
nd

in the third decade of life. They are the most common form of tumour in young men.
•• Risk factors include a history of testicular maldescent, a history of a contralateral testicular tumour and Klinefelter’s syndrome.
•• Seminomas metastasise mainly via the lymphatics and haematogenous spread is uncommon.
ha

•• Usually the patient presents with a painless testicular lump.


•• A sensation of heaviness can occur if the testis is two or three times its normal size, but only a minority of patients experience pain.
Some cases may simulate epididymo-orchitis and rarely some patients present with severe pain and acute enlargement of the testis
IC

because of haemorrhage into the tumour. Such cases can occasionally mimic testicular torsion.
•• In 1–2 per cent of cases, the tumour is bilateral at the time of diagnosis.
•• “Radical inguinal orchidectomy is the treatment of choice in all testicular tumour irrespective of the histological type and stage” (Manipal
PG

Surgery 4th/997)
Classification and pathology-Tumours of the testis are classified according to their predominant cellular type:
•• Germ cell tumours (90–95% per cent) (these include seminoma, embryonal cell carcinoma, yolk sac tumor, teratoma, and
choriocarcinoma);
•• Interstitial tumours (1–2 per cent) (these include Leydig cell tumours);
•• Lymphoma (3–7 per cent);
•• Other tumours (1–2 per cent).

166. Ans:  (b) Courvoisier law related to pancreatic head cancer, (c) Cholangitis presents with fever and jaundice
[Ref: Manipal Surgery 4th/566; PJM 20th/20;Schwartz 9th/1149;Davidson 22nd/ 937-38; Land B 26th/1115]

Cholangiocarcinoma
–– L and B 26th/1115
•• Early symptoms of cholangiocarcinoma are often non-specific with abdominal pain, early satiety, anorexia and weight loss commonly
seen. Symptoms associated with biliary obstruction (puritus and jaundice) may be present in a minority of patients.
•• In these patients, examination often demonstrates clinical signs of jaundice, cachexia is often noticeable and a palpable gall bladder
is present if the obstruction is in the distal common bile duct (Courvoisier’s sign).
“Stone in CBD: Intermittent pain, intermittent fever and intermittent jaundice are classical of stone in CBD”-Manipal Surgery 4th/566 Answers
“Cholangitis: The most common presentation is fever, epigastric or right upper quadrant pain, and jaundice. These classic symptoms, well &
Explanations
known as Charcot’s triad, are present in about two thirds of patients. The illness may progress rapidly with septicemia and disorientation,

265
PGI Chandigarh Self-Assessment & Review: 2017–2013

known as Reynolds’ pentad (e.g., fever, jaundice, right upper recommendations remain unchanged from previous versions
quadrant pain, septic shock, and mental status changes)”- but include in depth rationales from studies published since
Schwartz 9th/1149 the 2012 guidelines were released. The 2016 guidelines cover
Obstructive (Cholestatic) Jaundice the treatment of sepsis and septic shock in depth and include
–– Davidson 22nd/ 937-38 recommendations including fluid rescitation, vasopressor
•• Obstruction of the bile duct drainage due to blockage of the selection, antimicrobial therapy, mechanical ventilation,
extrahepatic biliary tree is characteristically associated with blood and blood products, insulin adminstration, DVT and
pale stools and dark urine. stress ulcer prophylaxis, and nutrition therapy”
•• Pruritus may be a dominant feature and can be accompanied
by skin excoriations. Peripheral stigmata of chronic liver Surviving Sepsis Guidelines 2012
disease are absent. –– jamanetwork.com/journal
•• If the gallbladder is palpable, the jaundice is unlikely •• “Notably, recombinant human activated protein C was
to be caused by biliary obstruction due to gallstones, completely omitted from the 2012 guidelines(in 2008
probably because a chronically inflamed stone-containing guideline it recommended for use in some selected
gallbladder cannot readily dilate. This is Courvoisier’s situations), prompted by the voluntary removal of the drug
Law, and suggests that jaundice is due to a malignant
by the manufacturer after failing to show benefit”-”- www.
biliary obstruction (e.g. pancreatic cancer).

e
the-hospitalist.org
•• Cholestatic jaundice is characterised by a relatively greater
•• “A higher SOFA score is associated with an increased

8/
elevation of ALP and GGT than the aminotransferases.
probability of mortality.The score grades abnormality
Table ( Davidson 22nd/ 937): Clinical features suggesting an by organ system and accounts for clinical interventions.
underlying cause of cholestatic jaundice However, laboratory variables, namely, Pao2, platelet count,

h,
creatinine level, and bilirubin level, are needed for full
Static or increasing jaundice: Carcinoma, Primary biliary cirrhosis, ar computation. Furthermore, selection of variables and cutoff
Primary sclerosing cholangitis values were developed by consensus, and SOFA is not well
Fluctuating jaundice: Choledocholithiasis, Stricture, Pancreatitis, known outside the critical care community”- jamanetwork.
ig
Choledochal cyst, Primary sclerosing cholangitis com/journal
Abdominal pain: Choledocholithiasis, Pancreatitis, Choledochal cyst •• Organ dysfunction can be identified as an acute change in
nd

Cholangitis: Stone, Stricture, Choledochal cyst total SOFA score ≥2 points consequent to the infection.
•• A SOFA score ≥2 reflects an overall mortality risk of
Abdominal scar: Stone, Stricture
approximately 10% in a general hospital population with
ha

Irregular hepatomegaly: Hepatic carcinoma suspected infection.


Palpable gallbladder: Carcinoma below cystic duct (usually pancreas) •• Patients with suspected infection who are likely to have
Abdominal mass: Carcinoma, Pancreatitis (cyst), Choledochal cyst a prolonged ICU stay or to die in the hospital can be
IC

Occult blood in stools: Ampullary tumour promptly identified at the bedside with qSOFA, ie, alteration
in mental status, systolic blood pressure ≤100 mm Hg, or
Table ( PJM 20th/20): Differential diagnosis of obstructive jaundice respiratory rate ≥22/min.
PG

(partial list) •• We recommend that administration of IV antimicrobials be


initiated as soon as possible after recognition and within 1
• Urine Bilirubin : Present
h for both sepsis and septic shock (strong recommendation,
• Urine Urobilinogen : Absent moderate quality of evidence; grade applies to both conditions).
• Stools Sterocobilinogen: Absent •• We recommend empiric broad-spectrum therapy with one
or more antimicrobials for patients presenting with sepsis or
167. Ans:  (b) qSOFA should be assessed in…, (c) Suggest against septic shock to cover all likely pathogens
the use of IV…, (e) Recommend empiric broad-… •• We suggest against using IV hydrocortisone to treat septic
shock patients if adequate fluid resuscitation and vasopressor
[Ref: Harrison 19th/1758; www.foamcast.org/2017/01/19; jamanetwork.
therapy are able to restore hemodynamic stability.
com/journal;Schwartz 9th/128-29]
•• We recommend against the use of erythropoietin for
As this Q is asked in November, 2016; we try to find answer of this treatment of anemia associated with sepsis (strong
Q according to 2012 Surviving Sepsis guidelines (Because latest, recommendation, moderate quality of evidence).
2016 guideline issued in Jan, 2017) •• We suggest against the use of IV immunoglobulins in
“The 2016 Surviving Sepsis guidelines produced by the Society patients with sepsis or septic shock (weak recommendation,
of Critical Care Medicine and the European Society of Intensive low quality of evidence).
Care Medicine have been released. Many of the guideline

Answers
&
Explanations

266
November  | 2016

Surviving Sepsis Campaign Recommendation Highlights


2012 2016
Sepsis Definition Systemic manifestation of infection + suspected Life threatening organ dysfunction caused by dysregulated
infection response to infection, No severe sepsis category
Severe sepsis: sepsis +organ dysfunction
Initial Resuscitation At least 30 cc/kg in first 3 hours
Crystalloid fluid (no recommendations on 0.9% NaCl vs balanced solution)
Albumin if patients require “substantial” fluids (weak)
Protocolized care including Use dynamic resuscitation markers (passive leg raise)
CVP Target MAP of 65 mmHg
ScVO2 Reassess hemodynamic status to guide resuscitation Normalize
Normalize lactate lactate
Vasopressors Target MAP of 65 mmHg
• Norepinephrine
• Epinephrine if not at target MAP OR vasopressin to reduce norepinephrine requirement
• Avoid dopamine in most patients

e
Steroids Only indicated for patients with septic shock refractory to adequate fluids and vasopressors
Antibiotics One or more antibiotics active against presumed Initial broad spectrum antibiotics (ex: vancomycin+ piperacillin-

8/
pathogen tazobactam)
Combination therapy (double coverage) for Against combined therapy (i.e. do not double cover
neutropenic patients and pseudomonas pseudomonas)

h,
May use procalcitonin to guide de-escalation
Source Control Achieve within 12 hours, if feasible ar Achieve as soon as medically and logically feasible
Ventilator 6 cc/kg tidal volume prone patients with severe ARDS (PF <150 in 2017 guidelines)
No recommendation Against high frequency oscillatory ventilation (HFOV)
ig
Weak recommendation for noninvasive ventilation in Unable to make recommendation on noninvasive ventilation
select patients with sepsis induced ARDS
nd

The New Surviving Sepsis Guideline 2016 (Earlier Guidelines Issued in 2012, 2008, 2004) is Being Rolled out in Nsw in January 2017:
Summary of New Recommendations emergencypedia.com
ha

•• Time to antibiotics within 60 minutes for patients with Sepsis (SOFA>=2), septic shock patients which essentially implies anyone
with qSOFA and/or organ dysfunction
•• Lactate measurement and targeting lactate reduction in those who have an elevated as a serial marker
IC

•• Blood cultures before antibiotics (unless there is going to be significant delay)


•• Early Empiric broad spectrum Antibiotic therapy for all sepsis patients (refer to local guidelines)
•• There is no mention of C-reactive protein anywhere in document
PG

•• Fluid Challenge for fluid resuscitation (rather than ‘drip‘ method)


•• What fluid? The usual crystalloids first, albumin next (NO ‘GEL’ and NO SPECIAL COLLOIDS)
•• Fluid Volume – up to 30ml/kg
•• What about Inotropes? – Norepinephrine – Vasopressin – Dobutamine OR Adrenaline in that order…
•• No routine use Steroids unless specific other indication(s)

168. Ans:  (a) Vagotomy- decreases acid…, (d) Billroth I gastrectomy has high rate…, (e) Highly selective vagotomy has low…
[Ref: Manipal Surgery 4th/471-75; L and B 26th/1036-40 ; Schwartz 9th/942]
“Typically, 15 to 30 minutes after a meal, the patient becomes diaphoretic, weak, light-headed, and tachycardic. These symptoms may
be ameliorated by recumbence or saline infusion. Crampy abdominal pain is not uncommon and diarrhea often follows. This is referred
to as early dumping, and should be distinguished from postprandial (reactive) hypoglycemia, also called late dumping, which usually
occurs later (2–3 hours following a meal), and is relieved by the administration of sugar”- Schwartz 9th/942

Billroth I Gastrectomy
–– L and B 26th/1038
•• The distal stomach is mobilised and resected in the same way as in the Billroth II gastrectomy. This resection should include the ulcer
that is usually situated on the lesser curve.
•• The incidence of recurrent ulceration after this operation is low, but it carries with it the morbidity and mortality associated with Answers
any gastric resection. &
Explanations

267

You might also like